Family Medicine EOR Prep

Réussis tes devoirs et examens dès maintenant avec Quizwiz!

a- The correct answer choice is atrial fibrillation. This patient has a history of hyperthyroidism, an irregularly irregular pulse, and an EKG finding of varying R-R interval and fibrillatory F waves in place of P waves, which is consistent with atrial fibrillation. Irregular rhythm, as well as lack of P waves, is particularly noted in leads II, III, and aVF. AF is typically treated with rate control in stable patients (not pharmaceutically) or cardioversion in an unstable patient. b- Atrial flutter would typically present with the classic "saw-tooth" appearance, most noted on leads II, III, and aVF. In addition, the pulse would typically be much faster. c- Supraventricular tachycardia is a rhythm that typically has a narrow QRS complex. However, aberrant conduction can lead to wide complex tachycardia that occasionally mimics ventricular tachycardia. This patient is not in ventricular tachycardia. The QRS complexes would be wide. e- Torsades de pointes, which literally means 'twisting of the points' in reference to the characteristic appearance, is an uncommon variant of ventricular tachycardia. It typically occurs as a complication of certain drugs, which cause a blockade of potassium channels, leading to a prolonged QT interval.

A 64-year-old woman with a past medical history of hyperthyroidism presents with new-onset palpitations. The patient states that she has intermittent palpitations, and she is worried about heart disease. She is currently asymptomatic and reports no chest pain or shortness of breath. An EKG is obtained. Vital signs reveal a T 98.6° F, BP 134/88 mm Hg, P 119 beats/min R 12/min. The pulse is noted to be irregular. The EKG is shown below. What arrhythmia does this patient most likely have? a. atrial fibrillation b. atrial flutter c. supraventricular tachycardia d. ventricular tachycardia e. Torsades de pointes

A patient comes to clinic for a growth on his forearm. He has had it for 6 years and is concerned that it may be enlarging. You observe a well circumscribed, pink, slightly pigmented, raised nodule about 1 centimeter in diameter. The center appears slightly ulcerated. What is the most likely diagnosis? a. basal cell carcinoma b. lipoma c. squamous cell carcinoma d. melanoma e. molluscum contagiosum

a- Basal cell carcinoma usually begins as a small indented nodule localized around hair follicles, typically in sun exposed regions of the skin. It typically begins to form an ulcerated center as it enlarges. This type of tumor rarely metastasizes but is locally invasive. It can be quite disfiguring. Excision is necessary to prevent its spread. b- Lipoma is a subcutaneous, soft round to oval mass of fatty connective tissue. The lesion is approximately 3-5mm in diameter and surrounded by a delicate capsule. The lipoma is normally a benign growth, but constant enlargement may cause compressive effects on surrounding tissues. Local excision is the usual mode of treatment. c- Squamous Cell carcinoma is also a slow growing tumor often found on sun exposed surfaces of the skin. This cancer does eventually metastasize. The lesion appears as a raised plaque which tends to ulcerate. Unlike basal cell carcinomas, this tumor originates in skin and squamous mucosa, which can result in lesions involving the skin, cervix, tongue, esophagus and lips. Histologically, this tumor may range from a more indolent type of tumor, containing keratin pearls of concentric lamellated keratin to a very anaplastic nature, where no pearls are evident. Squamous cell carcinoma metastasizes via lymphatics and then to the rest of the body. Excisional biopsy is required to determine the type of skin cancer and treatment necessary. d- Melanoma is typically a highly pigmented tumor of the skin, often seen in sun exposed regions. This tumor metastasizes widely and early on. The lesion may be a constellation of pigment colors, ranging from black to blue to red. The borders are frequently uneven. Tumors may arise from other organs including the meninges, throat (pharynx and larynx), and eye. Satellite nodules may also be seen distant from the main tumor site. Due to the aggressive character of this tumor, wide and deep incisions are necessary, along with tumor staging, if there is the possibility of metastasis. e- Molluscum Contagiosum is caused by a large DNA pox virus. This disease is typically seen in children and sexually active young adults. The lesion presents as a pearly, flesh colored papule approximately 2-10mm in diameter, often with a central umbilication. These lesions are often seen on the skin or mucous membranes. Treatment includes currettage and dessication or liquid nitrogen. Diagnosis is supported by the presence of intracytoplasmic inclusions in a KOH preparation of crushed tissue.

A 37 yo F presents with a 24-hour history of fever, right flank pain, vomiting, dysuria, and hematuria. A urinalysis reveals large amounts of red and white blood cells, as well as leukocyte esterase, and a subsequent urine culture was positive for >100,000 Escherichia coli. The patient denies having a history of renal calculi, and a bedside renal ultrasound does not show any stones or hydronephrosis bilaterally. She is diagnosed with a right-sided pyelonephritis and admitted for pain control, hydration, and IV antibiotic therapy. Her symptoms, including her fever, abate quickly, and she is discharged after 48 hours of being afebrile. Which prescription would she most likely receive upon discharge? a. ciprofloxacin 500 mg PO BID x 14 days b. Motrin 800 mg PO q 8 hours prn pain c. nitrofurantoin 100 mg PO BID x 7 days d. amoxicillin 500 mg PO BID x 14 days

a- Ciprofloxacin 500 mg PO BID x 14 days is the correct answer. Patients with pyelonephritis who are sick enough to be treated as an inpatient receive IV antibiotics until they have been afebrile for 24 - 48 hours. They also need to be able to tolerate oral hydration and oral medications before being discharged. Upon discharge, they will be given a prescription for antibiotics that will complete at least 2 weeks of antibiotic treatment. Ciprofloxacin has good coverage for E. coli urinary tract infections, and given at 500 mg PO BID x 14 days would be an appropriate choice as long as sensitivities from her culture showed Ciprofloxacin to have sensitivity. b- Motrin 800 mg PO q 8 hours prn pain is not the correct answer. While motrin is a good choice for patients who may still have some discomfort related to the pyelonephritis, it is not the most likely prescription for this patient to receive. She had become asymptomatic prior to discharge, so there would not be any reason to prescribe anything to help with pain or discomfort at that time. In addition, she is more likely to receive treatment for her infection than for pain. c- Nitrofurantoin 100 mg PO BID x 7 days is not the correct answer for several reasons. While nitrofurantoin is often an appropriate antibiotic for E. coli urinary tract infections, it does not achieve tissue levels reliable enough for pyelonephritis treatment. In addition, only 7 days of treatment does not add up to at least 2 weeks total of antibiotics. d- Amoxicillin 500 mg PO BID x 14 days is not the correct answer. While 14 days of antibiotics is a good length of time, amoxicillin does not have good coverage for E. coli, so it would not be a good choice to treat an E. coli related pyelonephritis.

A 37 yo F is reluctant to try an antidepressant for fear of becoming a drug addict. How should the physician respond? a. reassure her that antidepressants are not drugs of abuse because of oral administration, lack of immediate reward, and lack of tendency to cause tolerance b. Prescribe lower than recommended doses to avoid inducing a "high", explain to the patient she will not become addicted as long as she stops the drug after a few weeks c. give her the medication & refer her to a twelve step program d. Respect her concerns and treat her with psychotherapy alone e. Tapering the antidepressant to avoid withdrawal symptoms

a- Drugs can be screened in animals for abuse potential. Abusable drugs are rewarding (animals will self-administer them in preference to eating/drinking) and tend to induce both tolerance and withdrawal. The more rapid the onset of the sought for changes, the more likely the drug will be abused. Rapidity of onset is related to the particular drug taken and to the mode of administration. Drugs that are injected or inhaled are especially quick in action. Inhalation, in particular, bypasses first pass metabolism in the liver. Thus, IV use of heroin or snorting of cocaine (especially highly concentrated forms known as crack) quickly leads to abuse. Conversely, patients may be reassured that most prescribed medications, even psychotropics except benzodiazepines, are not abusable drugs and patients do not become addicted to them. Giving lower doses of antidepressants or prescribing them for a few weeks would be incorrect, since the onset of action is delayed up to 3 or 4 weeks and the drugs should be continued for at least 4 months. (Short courses followed by tapering would be correct for a benzodiazepine). Antidepressants, especially cyclic antidepressants, may precipitate a withdrawal syndrome if stopped abruptly, but this alone does not make them abusable.

A 28 yo F G0P0, and her husband present to your office after 1 year of infertility. Their histories elicit irregular menses in the woman and that the man is an avid cyclist. Apart from semen analysis in the man, what laboratory and/or diagnostic studies should be pursued in the woman to further evaluate this couple's infertility? a. fasting prolactin, TSH, FSH, LH, hysterosalpingogram b. FSH, T3, T4 c. TSH, LH, T3, T4 d. fasting prolactin, FSH e. fasting prolactin, TSH

a- Fasting prolactin, TSH, FSH, and LH should all be obtained in the female patient as her irregular menses more or less indicate a high probability of oligomenorrhea. It is also important to evaluate by hysterosalpingogram to look for evidence of uterine and/or tubal defects. It is also important to evaluate the male patient's semen to investigate male infertility related to oligospermia or hypomotility

A white male comes to the ED with the c/o pain in his right eye. He was whittling and felt like something went into his eye. This happened four hours ago. He has washed his eye with water, but it still feels funny. What is your working diagnosis? a. foreign body b. chalazion c. blepharitis d. conjunctivitis e. ectropion

a- Foreign Body sensation is most commonly due to corneal or conjunctival foreign bodies. Other causes are disturbances of the corneal epithelium and rubbing of eyelashes against the cornea (trichiasis). b- Chalazion is a mildly painful swelling of the eyelid margin, due to granulomatous inflammation, and usually resolves spontaneously with warm soaks and time. Multiple chalazion may be seen in diabetes, alcoholism, and malnourishment. Curettage may be required in extreme cases. c- Blepharitis may be seborrheic or ulcerative. Seborrheic (non-ulcerative) blepharitis is commonly associated with seborrhea of the face, eyebrows, external ears, and scalp. Inflammation of the eyelid margins occurs, with redness, thickening, and often the formation of scales and crusts, or shallow marginal ulcers. Ulcerative blepharitis is caused by bacterial infection (usually staphylococcal) of the lash follicles and the meibomian glands. d- Conjunctivitis can be caused by bacterial, viral, allergic, and irritant etiologies. Clients usually complain of red eyes, and a sticky or watery discharge. The type of discharge helps to determine the etiology-watery in viral, and sticky, green, or yellow, in bacterial conjunctivitis. Irritation is common, but severe pain and photophobia are not. Bacterial or viral conjunctivitis is usually self-limited, but it may be treated with a topical antibiotic, without steroids, such as sulfacetamide (10% 3-4 times/day). Topical aminoglycoside, gentamycin drops, should be reserved for more refractory disease. Allergic conjunctivitis may be effectively treated with a new class of non-steroidal topical, anti-inflammatory agents. Irritant conjunctivitis, including dry eyes, may be treated with topical, non-preserved lubricants. e- Ectropion is the outward turning of, usually, the lower lid occurring in older people. Surgery is indicated if ectropion causes excessive tearing, exposure, keratitis, or a cosmetic problem.

A 14 yo F presents for her school physical. On examination, you notice the posterior curvature of her thoracic spine to be very prominent and bulging backward. What type of deformity of the spine does she have? a. kyphosis b. scoliosis c. lordosis d. gibbus e. scoliokyphosis

a- Kyphosis is a posterior convex angulation of the thoracic spine as evaluated on a side view. Excessive and exaggerated angulation results in cosmetic problems, back pain, and cardio-respiratory problems. Kyphosis may be post-traumatic or non-traumatic type. Scheuermann's kyphosis, postural, congenital, and metabolic bone disease are a few of the non-traumatic causes of kyphosis. b- Scoliosis is the lateral curvature of the spinal column. This can be functional or structural when it compensates for other abnormalities. c- Lordosis is the anterior angulation of the spine in sagittal plane. It is the accentuation of the normal lumbar curve. It may be a compensation for other deformities such as kyphosis. d- Gibbus is a sharp angular deformity of the spine that could be due to a collapsed vertebra. The deformity may be protruding outward and may be caused by metastatic cancer or tuberculosisof the spine. e- Scoliokyphosis is the combined lateral and posterior curvature of the spine.

A 74 yo F presents because her children are concerned that that she might have dementia. She states that she is doing reasonably well, except that she sometimes sleeps less deeply and wakes up more often than she did several years ago. According to her children, she is slower than before, and her memory has been getting worse over the last 3 years; she has difficulties recalling the specific date of an event (although she can describe the event itself). She also has a great deal of trouble with names, but she can easily recognize people. She always says: "It is on the tip of my tongue, but..." Aside from hypertension that is under control, she does not have any other health problems. She has been a widow for about 10 years. Her older brother was diagnosed with dementia. Physical examination today is within normal limits for the age, and neurological examination is nonfocal. What should be your next diagnostic step? a. mini-mental status examination b. TSH & T4 c. MRI of the brain d. EEG e. Stanford-Binet Intelligence Test

a- MMSE is a brief 30-point screening test for cognitive impairment. In persons older than 70, the ability to memorize, acquire, and retain new information and recall names is diminished. Memory function may be disturbed in this way despite the relative retention of other intellectual abilities. It particularly applies to episodic memory, and it is manifested by difficulties with recalling names and/or specific dates. This disturbance is known as benign senescent forgetfulness or age-associated memory impairment. It progresses very little or not at all over a period of many years, and it does not interfere significantly with the person's work performance or activities of daily living. Your patient most likely has this sort of age related cognitive decline. MMSE can be used also as follow-up of her condition. b- You should perform a thyroid function test if you find out that a patient has dementia - slowness is a part of normal aging. c- MRI of the brain is indicated when structural changes are suspected; however, they are not likely in this patient due to her normal neurological examination. d- EEG is primarily used in the diagnosis of epilepsy, and it is sometimes used in the diagnosis of coma, encephalopathies, sleep disorders, and brain death. Your patient's sleep complaints are normal in advanced age. e- Stanford-Binet Intelligence Scales cannot be used to score adults.

A 49 yo Caucasian man presents with severe epigastric pain radiating to the back after a bout of drinking. Two weeks later, a repeat ultrasound showed a round, thin-walled hypoechoic lesion near the pancreas tail measuring 4 cm in its largest diameter and with some calcifications in its walls. MRCP visualized a communication between this cavity and the pancreatic duct. Endoscopic ultrasound detected an area of high-speed flow close to the gastric wall. Which of the following is the most appropriate next step in management? a. observation only b. perform angio-CT scan c. perform angiography with coil embolization d. refer for ERCP & tranpancreatic drainage of the cyst e. refer for a Puestow procedure

a- Pancreatic pseudocysts complicate about 10% of attacks of acute pancreatitis. A pseudocyst is formed when pancreatic fluid leaks and is confined by organs adjacent to the pancreas. Eventually, a fibrous wall forms around the collection. Most cysts regress spontaneously over a period of several weeks, but in some cases complications such as bleeding, abscess formation, and intractable pain may occur. Several interventional approaches — surgical, radiologic and endoscopic — have been tried. Endoscopic ultrasound has gained popularity in the management of pseudocysts, as it can identify complex cyst wall structures, in particular pseudoaneurysms, do fine-needle aspiration to rule out a neoplasm, and find a favorable spot for drainage. Ruling out a cystic pancreas neoplasm before endoscopic drainage is essential, as attempts at transgastric emptying of a neoplasm can have disastrous effects and compromise further surgical management of the neoplasia. Indications for intervention are intractable pain, expanding lesions, and infection. A frequently quoted study found that complications such as bleeding, abscess formation, or perforation rose sharply after 6 weeks of observation, and that intervention is warranted if a pseudocyst does not resolve in this period. However, more recent studies have challenged this assumption and proposed that observing asymptomatic pseudocysts for longer periods is safe. Pseudoaneurysms are found in 10% of pseudocysts. Signs suggesting a pseudoaneurysm include overt bleeding, sudden pseudocyst enlargement, and an abrupt fall in hematocrit. Their presence is a concern when drainage is considered and does not lead to a higher overall bleeding rate or constitute indication for intervention. c- Angiography has higher sensitivity and can perform therapeutics, but an angio-CT scan has adequate sensitivity for ruling out a pseudoaneurysm before proceeding with therapeutics. When intervention is warranted, angiographic coil embolization and operative resection can be performed.

A 2-week-old F infant is seen for her newborn well baby exam after a normal birth and delivery. She has been nursing well, has regained her birthweight and her development appears normal for her age so far. Physical examination is normal with the exception that ophthalmoscopic evaluation reveals a faint white reflex in her right eye. What is the most likely diagnosis? a. retinoblastoma b. retinoschisis c. phakomata d. retinitis pigmentosa e. retrolental fibroplasia

a- Retinoblastoma is the most common primary malignant intraocular tumor of childhood. It usually appears quite early in the first 5 years. Leukocoria, a white or Cat's eye reflex in the pupil is the most frequent finding. There may also be strabismus due to vision impairment. Ocular inflammation, intraocular hemorrhage, glaucoma or heterochromia iridis may be seen. On fundoscopic exam, the tumor may appear as a small to large white mass depending on its stage. Primary treatment includes enucleation, though smaller tumors diagnosed at an earlier stage may be amenable to newer alternative treatments such as cryotherapy and photocoagulation. Though leukocoria may be seen in retrolental fibroplasia or advanced stage of retinopathy of prematurity it is predominantly a disorder in preterm, low birthweight infants who received supplemental oxygen in the newborn period. These infants are susceptible due to the immaturity and subsequent damage of developing retinal vasculature. If the retina goes through various stages to ischemia and neovascularization, leukocoria may be seen representing retinal detachment and a subsequent membrane formation. b- Retinoschisis is a congenital disorder involving splitting of the retina into an inner and outer layer. Usually good vision is maintained. An elevation of the inner layer of the retina can be seen. c- Phakomata are retinal findings hallmarking hamartomatous disorders such as tuberous sclerosis. The distinctive ocular lesion is a yellowish multinodular cystic lesion arising from the retina or disc. Similar lesions can occur in neurofibromatosis. d- Retinitis pigmentosa is a progressive degeneration of the retina. It is characterized by pigmentary changes, arteriolar attenuation, some degree of optic atrophy and progressively deteriorating visual impairment. Granularity or mottling of the retinal pigment pattern or distinctive focal pigment aggregates can be seen fundoscopically.

A 60 yo M presents with a recent history of increased fatigue with mildly increased exertional dyspnea. Patient denies any significant past medical history but states that he had some heart problems as a child, though he was never clear as to what was the problem. On cardiac examination, you hear an early diastolic, soft decrescendo murmur with a high pitch quality, especially when patient is sitting and leaning forward. No thrill is felt. Based on this patient's presentation, you expect the patient to have __________. a. aortic regurgitation b. mitral stenosis c. tricuspid stenosis d. mitral valve prolapse e. pulmonic stenosis

a- The correct answer is aortic regurgitation, as it presents as a soft, early diastolic, high-pitched murmur heard best when sitting and leaning forward. It is often a result of rheumatic heart disease, which may be inferred by the patient's history b- Mitral stenosis is similar in presentation to tricuspid stenosis but is not affected by inspiration. c- Tricuspid stenosis presents as a diastolic rumbling murmur typically accompanied by a thrill heard louder on inspiration. d & e- Mitral valve prolapse and pulmonic stenosis are examples of systolic murmurs, in contrast to diastolic murmurs heard in this patient.

A 34 yo F presents after being found unresponsive in the bedroom of her home. She has a PMH of depression, and her mother found an empty bottle of amitriptyline by her bedside. Otherwise, she has no other medical or surgical history. The patient is a non-smoker and does not drink alcohol. On physical exam, her pulse is 138/minute, blood pressure is 80/60 mm Hg, temperature is 101.2°F (38.4°C), and respirations 6/minute. Her heart sounds are normal and she has thready pulses. Her breath sounds are normal, but with shallow effort. The abdomen is soft and nontender. Neurologically, she moves her limbs from painful stimuli. Her skin is flushed with no needle marks. Her CXR is normal, and the electrocardiogram demonstrates a wide complex tachycardia without ectopy. The patient is intubated and hyperventilated. What is the next appropriate step of the patient's management? a. administer sodium bicarbonate b. hemodialysis c. administer phenytoin d. administer IV lactated Ringer's solution e. administer physostigmine

a- The patient has taken amitriptyline, which is a tricarboxylic acid antidepressant (TCA); her set of symptoms are consistent with TCA toxicity. The MOAs of TCA are via anticholinergic effects, norepinephrine reuptake blockade, a quinidine effect, a sodium channel blocker, and peripheral alpha blockade. TCA-cardiotoxicity may be demonstrated on an electrocardiogram via sinus tachycardia, QRS complex prolongation > 100 milliseconds, right bundle branch block, ventricular tachycardia, ventricular fibrillation, and QT prolongation. Sodium bicarbonate is the DOC for the treatment of ventricular dysrhythmias and/or hypotension 2/2 (TCA) poisoning. b- Hemodialysis and hemoperfusion are not effective in TCA poisoning because small amounts of free TCA are present in the serum (mostly bound to serum proteins) & they are not recommended. c- In general, phenytoin is not efficacious with toxic seizures. There is debate as to whether there are any cardiovascular effects with phenytoin being a type 1B antiarrhythmic agent that counteracts the type 1A effect of TCA. d- Hyperventilation and hypertonic saline (e.g. lactated Ringer's) may also be useful, but clinical and experimental experience with these modalities is less extensive than with sodium bicarbonate. In patients with severe toxicity, bicarbonate needs to be given in order to achieve a serum pH of 7.50 to 7.55. Intermittent boluses of sodium bicarbonate are preferred to a constant infusion. e- Physostigmine in a hemodynamically-unstable patient is not the first-line drug; it may even be contraindicated. It should be considered if there are severe life-threatening anticholinergic effects.

An 8 yo boy presents with a 1-week history of wheezing and dyspnea on any exertion (with productive cough). On physical examination, bilateral rhonchi are heard. After a few days of treatment, spirometry is done on the patient. Total lung capacity (TLC) is 111% on spirometry. FVC% pre bronchodilators- 49%, post- 63% FEV1% pre bronchodilators 41%, post- 46% FEV1/FVC pre bronchodilators 49%, post- 55% What is the most likely diagnosis? a. asthma b. pneumonia c. pleural effusion d. tuberculosis e. kyphoscoliosis

a- The spirometry result in this patient shows reduction in both forced vital capacity (FVC) and forced expiratory volume in 1 second (FEV1), but FEV1 is more affected than FVC. Therefore, there is a decrease in the FEV1/FVC ratio below the predicted levels. The TLC is normal (80-120%). This concludes that this is an obstructive disorder and excludes the restrictive diseases listed. There is a marked improvement with bronchodilators. In reversible airway obstruction such as in asthma, there is a rise in the FEV1 and/or FVC by approximately 12% from pre- to post-bronchodilator testing. This reversibility is characteristic of bronchial asthma. The spirometry findings are not suggestive of restrictive lung diseases such as pneumonia, pleural effusion, kyphoscoliosis, and tuberculosis.

A previously healthy 34 yo M has had right flank pain radiating to his right groin for the past 3 hours. CT shows a 7 mm stone in his right ureter. What is the best option of treatment in this patient? a. lithotripsy b. opioid analgesics c. open surgical stone removal d. tamsulosin e. deflazacort

a- This patient has a 7mm ureteral stone and will likely require surgical intervention, such as lithotripsy, for this stone to pass. Peitrow and Micali note that 90 to 98% of stones <5 mm are likely to pass on their own, although sometimes >30 days are needed for this to occur. b, d, e- A variety of agents may assist in stone passage. Deflazacort, a steroid, decreases ureter edema and may facilitate stone passage through the ureter. Nifedipine and tamsulosin decrease ureter spasm, facilitating smoother stone passage through the ureter. Analgesics, particularly opioids and non-steroidals, decrease the pain associated with stones lodging in the ureter. c- Surgical interventions for stone removal include lithotripsy, a procedure which uses sound waves to break stones into smaller pieces which can be passed. Other procedures to assist in stone management include ureteral stenting, percutaneous nephrostomy tube placement, open surgical stone removal, and retrograde ureteral stone removal. However, since open surgical removal is an invasive procedure, lithotripsy is the first choice. Lithotripsy involves the usage of shock waves to crush stones in the renal calyx. It may be done as an outpatient procedure. Extracorporeal shock wave lithotripsy involves waves directed from outside; whereas intracorporeal shock wave lithotripsy consists of insertion of a percutaneous nephroscope and then crushing of the stones.

A 33 yo M presents with a 2-month history of coughing and a 2-day history of coughing blood. He has been losing weight and sweating at night. On physical examination, the patient appears wasted, tachypneic, has bronchial breath sounds on the right upper lobe, and crepitations on the left upper lobe and right mid zone. His direct sputum result comes back positive for acid-fast bacilli with Ziehl-Nielsen stain. His sputum is sent for culture, and treatment is started. Refer to the image. Retrobulbar neuritis is the predominant adverse effect of what drug? a. ethambutol b. streptomycin c. rifampin d. isoniazid e. pyrazinamide

a- This patient has all the complaints and symptoms of pulmonary tuberculosis (TB). Direct sputum examination by Ziehl-Nielsen stain also helps the diagnosis, but it is still not confirmatory. Sputum needs to be cultured to check what kind of mycobacterium is causing this disease. It is important to start the treatment as soon as the culture is sent. The standard treatment for adult respiratory/pulmonary TB includes a complete 6-month regimen comprising of 2 months initial phase with 4 drugs, which include rifampin, isoniazid, pyrazinamide, and ethambutol. This is followed by a 4-month continuation phase consisting of 2 drugs: rifampin and isoniazid. Irrespective of the bacteriological status of the sputum, this is the recommended standard treatment for respiratory tuberculosis (including isolated pleural effusion or mediastinal lymphadenopathy). The 4th drug, ethambutol, may be omitted in patients with a low risk of resistance to isoniazid. Ethambutol should be started in individuals who are known or suspected to be HIV positive, in those who have had previous treatment, and in immigrants and refugees of any ethnic group who are considered to have a significantly higher risk of resistance to isoniazid and other drugs. Like most medications, antituberculosis drugs also have some side effects. Since treatment is long-term, it is essential that patients are warned about and checked for side effects. If side effects are not explained well to the patient, it will decrease the compliance. The adverse effect of ethambutol is retrobulbar neuritis. The important side effects of anti-tubercular drugs are: INH: Hepatotoxicity, peripheral neuritis, cutaneous hypersensitivity, rarely can cause optic neuritis Rifampin: Hepatotoxicity, nephrotoxicity, red discoloration of the body fluids, 'Flu-syndrome,' and thrombocytopenic purpura Pyrazinamide: Hepatotoxicity, hyperuricemia Ethambutol: Retrobulbar neuritis Streptomycin: Nephrotoxicity, ototoxicity

A 26 yo F presents with inability to achieve intercourse with her partner since their relationship began 6 months prior. She states that she has been sexually active for 5 years, and penetration had been possible until recently; now, she cannot even insert a tampon due to the pain. Further history reveals that a year ago, her partner at that time forcefully made her have intercourse on several occasions. What is the next step in the workup of this patient to confirm the suspected diagnosis? a. gynecologic examination b. transvaginal ultrasound c. laboratory workup to expose the underlying medical condition d. referral to psychiatrist e. no further workup needed; the diagnosis is confirmed by history alone, so thus prescribe appropriate therapy

a- This patient is presenting with vaginismus, a contraction of the pubococcygeus musculature around the vagina, which results in painful or even impossible penetration. Vaginismus can either be primary (never achieving penetration) or secondary (previous penetration was possible). Many etiologies can elicit vaginismus, including an upbringing with rigid values against sexual intercourse, sexual trauma, fear of pregnancy, or questioning of sexual orientation. To confirm diagnosis, a gynecologic exam in which the same muscle spasm is elicited must be performed. However, the physician must be aware that the stimulus may only be due to coitus; thus, an absence of the muscle spasm does not rule out the diagnosis of vaginismus. Laboratory workup and ultrasound will show nothing definitive to confirm the diagnosis of vaginismus.

An 18 yo M presents with pain and deformity of his right dominant shoulder after a sudden jerking movement to the same from a wrestling competitor approximately 1 hour ago. He states he felt a clunking sensation when it happened. He was unable to continue wrestling and has pain with movement of the right shoulder. What diagnostic studies should be performed? a. internal & external rotated views of the humeral head b. AP & axillary or transscapular lateral radiographs c. MRI stat d. CT of the shoulder e- AP & internally rotated humeral view radiographs

b- AP & axillary lateral or transscapular lateral (Y-scapula) views should be obtained. It is imperative that a lateral view of the shoulder joint be obtained to assess the position of the humeral head. Posterior dislocations can be easily missed if the lateral view is not obtained.

A 55 yo M has had long term GERD symptoms. He has been on PPIs and has had fair control of his symptoms. Other PMH is unremarkable. He is a nonsmoker and drinks socially. Family history is significant for hypercholesterolemia in his father. Physical examination is unexceptional. An endoscopy a few years ago, revealed Barrett's esophagus by biopsy of the esophageal mucosa. He was recommended to have follow-up endoscopy every 2-3 years with mucosal biopsy. This screening was recommended to him because he is in danger of developing ____________. a. esophageal stricture b. adenocarcinoma of the esophagus c. squamous cell carcinoma of the esophagus d. achalasia of the cardia

b- Barrett's esophagus is an abnormality of the distal esophagus characterized by the replacement of normal squamous epithelium by metaplastic columnar epithelium with goblet cells, as a result of continuous inflammation most commonly from acid reflux. This is a premalignant condition with potential to develop into adenocarcinoma over time. Surveillance with screening endoscopy and mucosal biopsy is indicated to detect early onset of high grade dysplasia, which is then treated with esophagectomy. Failure to be compliant with the screening regimen may lead to adenocarcinoma of the esophagus, which has a poor prognosis as it presents late. a- Esophageal stricture is a distinct complication of long-standing GERD and may co-exist with Barrett's esophagus, since both have the same etiological factor, namely, reflux esophagitis. However, one does not necessarily predispose to the other, and in this patient absence of dysphagia makes the possibility of a stricture unlikely. c- Squamous cell carcinoma of esophagus is as frequent as adenocarcinoma, but risk factors are smoking and drinking alcohol, not Barrett's esophagus. d- Achalasia of the cardia is a motility disorder resulting from degeneration of ganglion cells in the esophagus and lower esophageal sphincter. Barrett's esophagus does not predispose to this condition.

A 28 yo F inquires when she should have her first mammogram. Her family history is significant for the death of her mother and grandmother due to breast cancer at the age of 32 years. Presently, the woman is asymptomatic. What is the recommendation for mammography for this patient? If you have a strong family history, the National Comprehensive Cancer Network recommends, starting at age 30, you get a: -Clinical breast exam every 6-12 months -Mammogram every year -Breast MRI every year When should you recommend a mammogram? a. when she is 30 years old b. immediately c. when she is 32 years old d. when she is 34 years old e. when she is 36 years old

b- Breast cancer is recognized as both an increasingly common disorder and a potentially hereditary disorder. Furthermore, early detection will provide a woman with her best chance at a cure of breast cancer. Given the patient's genetic burden of having both her mother and her grandmother die of metastatic breast cancer at the relatively young age of 32 years, it is reasonable to provide her with immediate and periodic mammography combined with instruction in breast self-examination. Women at high risk require a more aggressive schedule, frequent examinations, and other procedures such as ultrasound and MRI. American Cancer Society Guidelines Women ages 40 to 44 should have the choice to start annual breast cancer screening with mammograms if they wish to do so. Women age 45 to 54 should get mammograms every year. Women 55 and older should switch to mammograms every 2 years or can continue yearly screening. Screening should continue as long as a woman is in good health and is expected to live 10 more years or longer. The ACS does not recommend clinical breast examination for breast cancer screening among average-risk women at any age ACOG continues to recommend adherence to its current guidelines, which include the following: Screening mammography every 1-2 years for women aged 40-49 years Screening mammography every year for women aged 50 years or older

A 35 yo F presents with vaginal discharge. She has a history of itching and white discharge. She is sexually active with her husband only. She takes oral contraceptive pills for contraception, and she has never been diagnosed with an STD. On examination, the vagina is hyperemic and covered with white cottage cheese-appearing discharge. What is the most likely diagnosis? a. bacterial vaginosis b. candida vaginitis c. herpes simplex d. trichomonas e. gonorrhea

b- Candida vaginitis is the colonization of vagina and vulva by the fungus Candida albicans. It is favored by the acidic media and usually flourishes in patients on broad-spectrum antibiotics, patients who are diabetic, patients taking corticosteroid medications, those who are pregnant, and those who are immunocompromised. Another factor is increased wetness of the area, especially in combination with certain occlusive clothing. It usually presents with severe pruritus, erythema, and a curd-like discharge. Microscopic examination with KOH preparation directly or after culture on Nickerson's medium can be done. Under microscope, either from the direct swab or the culture, there is an appearance of a characteristic shape (hyphae and spores). Treatment with miconazole is usually effective. Some physicians add cortisone cream to relieve itching. a- Bacterial vaginosis, formerly known as non-specific vaginitis, is caused by a mixture of microorganisms, of which Haemophilus or Gardnerella are the most common and well known. It causes malodorous (fishy) discharge and is treated with metronidazole or clindamycin. Wet mount preparation with saline shows clue cells, which are epithelial cells covered with bacteria. Culture is not effective and not used. c- Herpes is caused by infection with the herpes simplex virus. Patients present with severe pain and vesicles. Treatment is with acyclovir. d- Trichomonas is caused by Trichomonas vaginalis, a parasite with flagella that causes copious vaginal discharge. Treatment with metronidazole is recommended. It can be seen under microscope (using wet mount preparation) as a motile parasite with flagella. e- Gonorrhea is a sexually transmitted disease caused by Gonococci (Gram-negative diplococci). It is characterized by discharge, fever, and sometimes joint pain and conjunctivitis. Treatment of choice is either ceftriaxone 125 mg IM or cefpodoxime 400 mg orally as a single dose.

A 53 yo F presents with nervousness, a tremor, and irritability. She indicates that she has lost some weight over the last few weeks even though her appetite has increased. She also says that she is feeling feverish. You suspect that she may have a thyroid condition and order the appropriate labs. The lab results are as follows: TSH 7.8 (0.5 - 5.5 mU/L) T4 4.2 (0.8 - 1.8 ng/dl) T3 5.3 (1.4 - 4.2 pg/ml) a. thyroid storm b. pituitary adenoma c. Hashimoto's thyroiditis d. Graves' disease e. iodine deficiency

b- The clinical picture is of a pituitary adenoma. The diagnosis is suggestive by clinically evident hyperthyroidism with elevated T4 and T3 and elevated serum TSH levels. a- Untreated hyperthyroidism may progress into a thyroid storm. Patients may present with tachycardia, fever, agitation, nausea, vomiting, diarrhea, and restlessness or psychosis. c- Hashimoto's thyroiditis would generally produce elevated TSH and decreased T4 and T3 levels. d- Graves' disease would produce decreased TSH and elevated T4 and T3 levels. e- In iodine deficiency or goiter, there is diffuse enlargement of the gland as well as elevated TSH and decreased T4 levels.

A 53 yo M presents with chest pain. He has a PMH of hypertension and diabetes mellitus. The pain is located in the middle of his chest and radiates to his jaw. The pain began about 20 minutes ago, and he rates the pain as a 10 on a 0 - 10 point scale, with 10 being the worst pain he has ever felt. He has had 3 similar episodes, but they have always resolved after 5 minutes or so of rest. He has smoked 1 pack of cigarettes a day for the past 36 years. He drinks 2 or 3 beers on Friday nights. Review of systems (ROS) is positive for diaphoresis, acute dyspnea, and impending doom. ROS negative for fever, chills, and malaise. Physical exam shows an obese, middle-aged man in moderate distress. BP 126/80, pulse 100, respirations 26. Heart and lung exams are normal, except for tachycardia and tachypnea. He has no pedal edema. What aspect of the patient's history is the largest risk factor for an acute myocardial infarction? a. cigarette smoking b. diabetes mellitus c. HTN d. alcohol use

b- DM is an independent risk factor for atherosclerosis; the risk of MI in a diabetic is the same risk as someone w/o diabetes who has had a previous MI & the risk of death from cardiac events is also the same amongst the 2 groups; patients with DM should be advised to stop smoking & aggressively control other risk factors (e.g. glucose, HTN, DLD) to reduce the risk of ischemic heart disease d- alcohol consumption may have a protective effect as people who drink moderate amounts of alcohol reduce their risk of CV events

A 25 yo M residing in Thailand presents with high-grade fever, dull, frontal headache, malaise, anorexia, and vague abdominal discomfort of 7 days duration. He had mild diarrhea, dry cough, and myalgia. On examination, his temperature was 39 degrees C. His pulse was 65 per minute. He had a coated tongue, tender abdomen, and a soft palpable spleen. Clinical diagnosis was Enteric fever. Appropriate clinical samples were sent for culture and serology. What laboratory data would be helpful in making a definitive diagnosis? a. serum titer of S. typhi H agglutinins 1:160 b. blood culture positive for S. typhi c. stool culture positive for S. typhi d. serum titer of S. typhi O agglutinins 1:80 e. urine culture positive for S. typhi

b- Definitive diagnosis depends on a positive blood culture for Salmonella typhi. The term Enteric fever includes typhoid caused by Salmonella enterica serotype, Typhi (referred to as S.typhi), and paratyphoid caused by Salmonella paratyphi A, B, and C. Isolation of S.typhi from stool and urine samples helps diagnosis, but could be positive in carriers as well. Due to various factors, the interpretation of agglutinin titers detected in a single serum sample is difficult. In enteric fever, the infection is acquired by ingestion. After reaching the small intestine, the bacteria invade the mucosa reaching the mesenteric lymphoid follicles and the draining mesenteric lymph nodes. Some pass on to the reticuloendothelial cells of the spleen and liver. Here, salmonellae are able to survive and multiply within the mononuclear phagocytic cells. Depending on the virulence of the bacteria and host response, bacteria are released into the blood stream and become widely disseminated. The bacilli are seeded in the liver, spleen, gall bladder, bone marrow, and Peyer's patches of terminal ileum, where further multiplication occurs. The lungs and kidneys may be involved. Towards the end of the incubation period (7-14 days), massive bacteremia occurs, leading to the onset of clinical symptoms. During the first week of clinical disease, Salmonella can be isolated from blood culture in 60-80% of patients. As the humoral and cellular response of the host come into action, the chances for isolation from blood gradually decrease. Since multiplication of organisms occurs in the intestinal lymphatic tissue and in the gall bladder, Salmonellae are excreted in feces. The stool culture positivity rate is more during the second and third week of illness. When salmonellae localize in the kidneys, they are excreted in urine, though infrequently and in small numbers. It is possible to isolate the organisms from urinary deposits by inoculating enrichment media. The sensitivity of the blood culture is reduced by prior administration of antibiotics. In such patients, bone marrow culture is very useful for the isolation of the bacteria. Positive results are obtained in 80-95% of patients with typhoid, and are not affected by the duration of the illness and/or prior antibiotic therapy. DNA probes and PCR-based tests have been developed to detect S.typhi directly in blood, though these are not available for use in many areas where typhoid is common. Prevalence of multi-drug-resistant (MDR) S.typhi (resistant to chloramphenicol, ampicillin and trimethoprim/sulfamethoxazole) is a major problem faced by countries where typhoid is endemic. The resistance is plasmid mediated. Third generation cephalosporins and fluoroquinolones are recommended for treatment of infections by MDR strains. Recently, the emergence of strains with reduced susceptibility to fluoroquinolones has been noticed. This is chromosomally acquired resistance, and is largely due to mutations in the gyrA gene. Development of resistance to ceftriaxone has been reported, though is rare in occurrence. In making appropriate decisions regarding the choice of an antibiotic for the treatment of typhoid, susceptibility testing of the isolate is very important. a- H agglutinins are influenced by previous infection and past immunization against typhoid. They persist for a longer period than O agglutinins. Though the Widal test is still used for diagnosis of enteric fever in countries where the disease is endemic, the test faces controversies. Widal test results are influenced by a variety of factors, including previous antibiotic therapy, previous immunization with Salmonella antigens, and cross-reacting antibodies to other infectious agents. Therefore, the interpretation of the test should be done with caution. Information on the distribution of agglutinin levels in normal sera, in different geographical areas, may help to assess the significance of the test results. During recent years, rapid tests such as Dot blot and DOT enzyme immunoassay for serodiagnosis of enteric fever have been developed and evaluated. The results are promising, and these tests are reported to be of use in regions where typhoid is endemic and laboratory facilities are limited. c- A positive stool culture is helpful in diagnosis, though it is not always confirmatory. Fecal carrier state is common after clinical as well as sub-clinical infection by Salmonella typhi. In fecal carriers, salmonellae persist in the gall bladder and the biliary tree, and they are excreted in the feces intermittently. Chronic carrier state is seen more commonly in patients with cholelithiasis. Demonstration of Vi agglutinins in the serum and culture of stools and bile samples are useful for detection of carriers. d- O (somatic) and H (flagellar) agglutinins against Salmonella typhi and paratyphi are tested by Widal agglutination assessment. A single serum sample test result has no diagnostic significance. Serum agglutinins usually appear by the end of first week and rise sharply during the second and third weeks of Salmonella infection. A rise in titer of agglutinins is indicative of active infection, and it is to be demonstrated by testing at least 2 samples of serum that are taken at an interval of 7-10 days. Demonstration of a 4-fold rise in titer of agglutinins against H and O antigens of S.typhi can be helpful in diagnosing typhoid during later stages of the disease, when blood culture is often negative. Though a high titer of O agglutinins could be suggestive of recent infection, the possible presence of cross-reacting antibodies limits the utility of this test. e- Urine culture can be positive in urinary carriers as well. Urinary carriers are less frequent than fecal carriers. The carrier state is generally associated with some abnormality of the urinary tract, such as calculi or schistosomiasis.

A 57 yo F with a 15-year history of T2DM presents for follow-up of her diabetes. Her spot albumin/creatinine ratio was 100 mg/g 4 months ago and was confirmed at 100 mg/g yesterday. Her urinary analysis shows no cells, casts, or blood. Her creatinine is 0.7 mg/dl and her estimated glomerular filtration rate is 95 ml/min/1.73 m2. What medication(s) should you prescribe to help prevent her progression from micro to macroalbuminuria and to help prevent a progressive decline in glomerular filtration rate? a. calcium citrate b. lisinopril c. calcium carbonate d. potassium chloride e. sodium bicarbonate

b- Lisinopril, an ACE Inhibitor, should help decrease albuminuria, prevent progression of diabetic kidney disease from micro to macroalbuminuria, and prevent a decline in glomerular filtration rate. This class of medications has been studied extensively for these purposes. Angiotensin II receptor blockers (e.g., irbesartan) may also reduce urinary albumin to normal levels. Monotherapy with either of these classes of medications should be attempted first in patients with microalbuminuria. This will test tolerance, effectiveness, and adverse reaction such as hyperkalemia. For patients with greater degrees of albuminuria (e.g., 1 g/day), poor response to monotherapy and blood pressure control, and no hyperkalemia associated with therapy, combination therapy should be considered. Combination therapy with both ACE inhibitors (e.g., lisinopril) and angiotensin II receptor blockers (e.g., irbesartan) is used to treat both diabetic and non-diabetic kidney disease. These medications act on different parts of the renin angiotensin system. In combination, irbesartan could block the effect of angiotensin produced by non-ACE pathways and lisinopril could block the production of angiotensin stimulated by irbesartan in a negative feedback system; however, combination therapy is usually preceded by monotherapy. Although combination therapy is currently being used in both diabetic and non-diabetic kidney disease, this therapy still being researched. It is unknown whether monotherapy alone is sufficient. It does not appear to be sufficient in all patients, particularly those with persistent micro and macroalbuminuria despite monotherapy. a & c- Calcium carbonate and calcium citrate are used to treat the hyperphosphatemia found commonly in patients with chronic kidney disease. They will not alter the progression of diabetic proteinuria. d- Potassium chloride is a mineral used in the management of hypokalemia. We have no knowledge of hypokalemia in this patient and no indication to use potassium supplements. Potassium is not known to alter the progression of diabetic nephropathy. In fact, this woman is more likely to be hyperkalemic due to her diabetes, which can cause hyporeninism, hypoaldosteronism, and hyperkalemia. Giving potassium to patients already at risk for hyperkalemia is not advised, as hyperkalemia may provoke cardiac arrhythmias. e- Sodium bicarbonate is used to treat the metabolic acidosis that commonly occurs secondary to chronic kidney disease. Diseased nephrons eliminate acids poorly, allowing them to build up in the blood stream. Sodium bicarbonate buffers these acids. Sodium bicarbonate is not known to significantly alter progression of proteinuria or decline in glomerular filtration rate.

A 43 yo hypertensive, diabetic Native American woman is scheduled for a follow-up visit today. She says that her father recently passed away after having a heart attack. She is extremely worried and wants to know what she can do to reduce her risk of cardiovascular disease (CAD). You explain to her that that the risk factors for CAD are classified as either non-modifiable or modifiable. What is the strongest non-modifiable risk factor for CAD in this woman? a. her age b. her ethnicity c. HTN d. diabetes e. her gender

b- Native Americans are at higher risk of CAD compared to Caucasians. The risk factors for CAD may be classified as non-modifiable and modifiable. c & d- The modifiable risk factors include diabetes, hypertension, smoking, body weight, and blood lipid levels. A sedentary lifestyle and fat- or salt-rich diets are also considered risk factors. The non-modifiable risk factors include increasing age (>45 for men and >55 for women), male gender, positive family history of CAD, and ethnicity. African-Americans, Native Americans, Mexican Americans, and some Asian groups are at a greater risk than Caucasians, which is partly due to higher incidences of hypertension, diabetes, and obesity among these groups.

A 43 yo M presents for follow-up of poorly controlled T1DM, which was diagnosed 32 years ago. What change on his funduscopic examination would indicate a need for urgent referral to an ophthalmologist? a. flame-shaped hemorrhages b. neovascularization c. cotton wool spots d. blot hemorrhages e. microaneurysms

b- Neovascularization is the hallmark of proliferative diabetic retinopathy. New vessels can appear at the optic nerve and the macula as a result of retinal hypoxia. They are susceptible to rupture, resulting in vitreous hemorrhage, retinal detachment, and blindness. Proliferative retinopathy requires urgent referral to an ophthalmologist and is usually treated with pan retinal laser photocoagulation. The risk of developing diabetic retinopathy is related to the extent of glycemic control and the duration of diabetes. It is classified as nonproliferative and proliferative. c, d, & e- Blot hemorrhages, cotton wool spots, and microaneurysms are indicative of nonproliferative diabetic retinopathy, which is usually seen 10 to 20 years after the onset of diabetes. Nonproliferative retinopathy does not always progress to proliferative retinopathy, but if it becomes extensive, it can result in retinal ischemia, which increases the likelihood of proliferative disease. a- Flame-shaped hemorrhages are indicative of hypertensive retinopathy.

A 24 yo M presents to you with an acute otitis media with serous otitis in the right ear. You perform the Weber and Rinne tests. Which of the following results would you most likely expect to find? a. Weber - sound is heard louder in right ear, Rinne - air conduction exceeds bone conduction in right ear b. Weber - sound is heard louder in right ear, Rinne - bone conduction exceeds air conduction in right ear c. Weber - sound is heard louder in left ear, Rinne - bone conduction exceeds air conduction in right ear d. Weber - sound is equal in both ears, Rinne - bone conduction greater than air conduction in right ear e. Weber - sound is heard louder in left ear, Rinne - air conduction exceeds air conduction in right ear

b- Otitis media and serous otitis are examples of causes of conductive hearing loss. When a conductive hearing loss exists, the Weber test will result in the appearance of a louder sound in the affected ear and the Rinne test will result in bone conduction exceeding air conduction in the affected ear. The other answers are incorrect because in a sensorineural hearing loss, the Weber test results in a louder sound in the unaffected ear and the Rinne test will result in air conduction exceeding bone conduction in the affected ear.

Deficiency of what factor may predispose a person to recurrent thrombosis? a. Factor VII deficiency b. von Willebrand factor deficiency c. Factor VIIIC deficiency d. protein C deficiency e. platelet deficiency

d- Protein C is a potent anti-coagulant. Both protein C and co-factor protein S are vitamin K dependent peptides. Protein C, which is produced by the liver, is a powerful naturally occurring anti-coagulant and is activated by thrombin. Thrombin (a powerful pro-coagulant) activates protein C after binding with a factor known as thrombomodulin. Activated protein C digests factor VIIIA and Va. Protein S is produced by endothelial cells and acts as a co-factor for protein C.

A 57 yo F presents with a 1-year history of chronic cough. The cough produces large volumes of grossly purulent sputum. She has a history of recurrent respiratory infections; they resulted in 5 hospitalizations in the past year. She also had similar complaints during the previous year. Shortness of breath limits her daily activity considerably. Upon pulmonary examination, bilateral breath sounds are audible, with inspiratory and expiratory crackles at the lung bases. CXRs reveal increased lung volumes, flattened diaphragm, and tram track lines. What is the initial diagnosis? a. pulmonary fibrosis b. chronic bronchitits c. pulmonary edema d. emphysema e. asthma

b- The clinical picture is suggestive of chronic bronchitis. Chronic bronchitis is defined by a clinical history of productive cough for 3 months of the year for 2 consecutive years. Smoking is the leading cause. The principle pathologic feature is airway injury and narrowing, hypertrophy of the airway mucous glands, infiltrate of inflammatory cells, and loss of ciliated epithelium. The cough produces thick, often purulent sputum because of the ongoing local inflammation and the high likelihood of bacterial colonization and infection. The increased mucous production and defective mucociliary escalator function leads to inspiratory and expiratory crackles. On imaging, common findings are hyperinflation of lung volumes, depressed diaphragm, and parallel linear densities known as tram track lines. a- Pulmonary fibrosis is a restrictive lung disease with the clinical features of progressive dyspnea. It is typically accompanied by a dry, persistent hacking cough. c- Pulmonary edema may present with mild exertional dyspnea or a nonproductive cough, although a frothy or blood-tinged sputum may be seen. d- A productive cough is not a clinical manifestation of emphysema. e- Asthma produces wheezing lung sounds. They are not heard in this patient.

A 65 yo M presents with sudden onset of brief episodes of blindness in his right eye, with complete recovery before the next episode. The event is described as a shade coming down across his field of vision, and it is not painful. What is the most common underlying condition leading to the event described above? a. sickle cell disease b. carotid stenosis c. migraine headaches d. lower extremity DVT e. acute angle-closure glaucoma

b- The diagnosis for this case is amaurosis fugax. The most common cause of amaurosis fugax is ipsilateral carotid stenosis, leading to carotid emboli that lodge in the retinal arteries, causing temporary blindness until the emboli dissolves. Screening is performed through carotid Doppler ultrasonography, CT, or MR angiography. Treatment includes low-dose aspirin or other anti-platelet therapy, along with treatment of underlying cause. Migraine headaches, sickle cell disease, and acute angle-closure glaucoma are all less common causes of amaurosis fugax. Lower extremity deep venous thrombosis is not a likely source for emboli causing amaurosis fugax.

A 4 yo boy has velvety lax skin, hyperextensible joints, and mitral valve prolapse. What is the most likely diagnosis? a. osteogenesis imperfecta b. Ehlers-Danlos syndrome c. epidermolysis bullosa d. Marfan syndrome e. achondroplasia

b- There are several different types of Ehlers-Danlos syndrome (EDS). The clinical vignette above describes the classical, type I variant. The velvety, distensible skin, hyperextensibility of many joints, mitral valve prolapse, and hernia are all associated with the type I variant. Although the exact nature of the biochemical defect is just now being sorted out, all types of EDS are due to mutations in collagen genes. In recent experimental studies, a mutation in the gene encoding the α1(V)-chain of type-V collagen, was identified. Type- V collagen is associated with type-I collagen and is probably involved in assembly of type-I collagen fibers. Type-I collagen is abundant in ligaments of joints and in the dermis, thus the characteristic and unusual symptoms in the skin and joints.

A 71 yo M presents with the inability to actively raise his left non-dominant arm to retrieve plates from the kitchen cabinet. This began a month ago after his shoulder pain improved. He had a history of pain in that shoulder for over 6 months that kept him from sleeping on the left side and the pain would wake him often. There was no specific injury he can recall although he felt a pop a month ago while taking out the trash. What is the diagnostic study of choice if surgery is indicated? a. plain films b. MRI c. ultrasound d. CT without contrast e. electromyelogram

b- This elderly man probably has a torn rotator cuff. If surgical treatment is being considered, MRI is the imaging study of choice because it can provide additional information on the status of the muscle and the size of the rotator cuff tear. e- Electromyelogram is used to assess the nerves. c- Ultrasound doesn't give as clear a picture of the tear due to the bony interference. a- Plain films can assess only the bony picture.

A 37 yo M with known stage 3 CKD 2/2 DM presents with fatigue. His blood sugars are acceptable; his mood is euthymic, and he is sleeping and eating well. He does not smoke, and he has no known toxic environmental exposures. Work up shows normochromic normocytic anemia, with a hemoglobin of 11 g/dl, which is decreased from 12 g/dl 6 months ago. Medications include lisinopril, furosemide, and insulin. Labs: Iron 50 ug/dl (normal 50-150 ug/dl) Ferritin 200 ng/ml Ratio soluble transferrin receptor/log ferritin Reduced Reticulocytes 0.25% Platelets Normal Total WBC 5x10^3/mm3 B12 levels, serum Normal What is the most likely diagnosis? a. pernicious anemia b. anemia of chronic disease c. iron deficiency anemia d. thalassemia e. myelodysplasia

b- This patient most likely has anemia of chronic disease secondary to his chronic kidney disease. Anemia of chronic disease is characterized by normochromic normocytic anemia, decreased reticulocytes, low iron levels, normal-to-increased ferritin levels, and low soluble transferrin receptor to log ferritin ratios (Weiss). It is the second most common cause of anemia after iron deficiency anemia (Weiss), and it may complicate a variety of diseases, including infections, cancers, autoimmune diseases, chronic kidney disease, and transplant rejection. In anemia of chronic disease, high cytokine production leads to decreased gastrointestinal iron absorption, decreased regulation of iron transporters, decreased erythropoietin production, and decreased proliferation and differentiation of erythroid precursor cells. The result is normochromic normocytic anemia. Patients may complain of fatigue and decreased quality of life. Treatments include erythropoietin and iron to support hemopoiesis. Treatments may correct anemia and improve symptoms, but they will unlikely correct chronic kidney disease. a- Pernicious anemia is due to B12 deficiency. Unlike anemia of chronic disease, macrocytosis is noted. Low iron levels are unexpected. Poor gastrointestinal absorption and dietary intake may cause this deficiency, and treatments include oral supplements and intramuscular Vitamin B12 injections. c- Iron deficiency results from the loss of blood from the gastrointestinal tract, menstrual flow or wounds, or from impaired iron absorption and processing. Low blood iron levels, microcytic hypochromic anemia, low ferritin levels, and high ratios of transferrin receptor to log ferritin are expected (Weiss). In anemia of chronic disease, low iron levels are also noted, but they are accompanied by other changes, as noted above. d- Thalassemias are generally inherited conditions whereby abnormal globin synthesis leads to microcytic hypochromic and abnormal shaped red blood cells. They may be diagnosed by electrophoresis in early age in anemic, failing to thrive patients. Smoking and carbon monoxide exposure may also lead to hemoglobinopathies and decreased ability of red cells to carry oxygen (Harrison's chapters 90, 91). This patient's non-pediatric age and lack of environmental exposures makes thalassemias an unlikely cause of his anemia. Additionally, normal to high iron levels and transferrin saturations would be expected in thalassemia e- Like anemia of chronic disease, myelodysplasia (MDS) is characterized by normochromic cells and low reticulocyte counts. Normocytemia to macrocythemia may be seen. Pancytopenia is more common in myelodysplasia. In this vignette, the normal platelet and white blood cell counts make myelodysplasia less likely.

A 37 yo M is admitted to the hospital with progressive SOB, fever, and worsening cough. The patient had been in good health until 2 months ago, when he began losing weight. This was associated with anorexia, intermittent diarrhea, night sweats, and then a nonproductive cough. He had lost more than 20 pounds by the time he was admitted to the hospital. His past medical history is unremarkable. He has been divorced for 5 years, and he has 1 child. He is employed as a medical equipment salesman, traveling extensively in the Midwest. He admits to drinking alcohol in large amounts on weekends, but he denies tobacco and intravenous drug use. He gives history of a previous homosexual encounter. Physical examination shows that the chest was normal to percussion and clear by auscultation, except for a few scattered rhonchi. The heart is normal except for tachycardia. The abdomen is soft with normal bowel sounds. Genitalia are normal; however, there is a painful 2 cm ulceration at the anal verge. The neurologic exam is unremarkable. Chest radiological findings show diffuse bilateral interstitial infiltrates. Arterial blood gases on room air show pO2- 57 mmHg, pCO2 31 mmHg, and pH 7.45. His alveolar-arterial O2 gradient is 55 mmHg. Bronchoalveolar lavage fluid with lung biopsy shows the presence of cysts. Sputum cytology is negative for acid-fast bacilli. What is the most likely diagnosis? a. secondary syphilis b. pneumocystis pneumonia c. AIDS-related TB d. lung cancer e. Legionella pneumonia

b- Whenever a young patient presents with fever, progressive exertional dyspnea, hypoxia, and loss of weight, the possibility of Pneumocystis jiroveci pneumonia complicating acquired immunodeficiency syndrome (AIDS) should be considered, especially when diffuse interstitial infiltration (or patchy shadows) are found on chest radiological study. The history of a homosexual encounter favors this diagnosis. This should be followed by tests to confirm HIV. The increased alveolar-arterial O2 gradient indicates severe respiratory dysfunction. Bronchoalveolar lavage with lung biopsy is an appropriate early step in his evaluation. Finding pneumocystic cysts in the alveolar lavage is a confirmatory diagnosis for Pneumocystis jiroveci pneumonia. Treatment is based on the alveolar-arterial O2 gradient, which is considered mild when the value is less than 35mmHg, moderate when it is 35-45mmHg, and severe disease when more than 45mmHg. The mainstay of treatment is given intravenously or orally. Combined therapy of trimethoprim-sulfamethoxazole and corticosteroids is necessary in the treatment of a severe case of pneumocystis in AIDS. Administration of corticosteroids helps to prevent respiratory failure and death in AIDS patients. When Pneumocystis jiroveci Pneumonia is found in the absence of underlying immunosuppression from malignancy or drug, the patient fulfills the definition of AIDS.

A 13 yo M presents with left shoulder pain and deformity after a fall from his skateboard while attempting a half-pipe maneuver an hour ago. He heard a "cracking" sound and was unable to continue skateboarding. He has pain with movement of the arm in any direction. On exam, you note tenderness over the superior central shoulder with a palpable deformity and "tenting" of the skin. What is his most likely diagnosis? a. frozen shoulder b. clavical fracture c. rotator cuff tear d. glenohumeral dislocation e. AC separation

b- With a superior central deformity and tenderness, along with tenting of the skin, the most likely diagnosis is clavicle fracture. a- Frozen shoulder is a slow, progressive chronic problem and not an acute injury. c- Rotator cuff tear does not have acute deformity. d- Glenohumeral dislocation would have shoulder contour deformity. e- AC separation would have deformity laterally on the shoulder.

A 44 yo F presents with a 4-month history of dull, aching heaviness sensation in her proximal right leg. She notes that this sensation is provoked by extended periods of standing and walking, and is relieved when she lies in a recumbent position. Her PMH is remarkable for pregnancy 4 times, the most recent being approximately 2 years ago. She denies a history of smoking, trauma, injuries, fever, chills, chest pain, shortness of breath, hemoptysis, cough, skin changes and coolness, and peripheral edema. Her physical exam reveals several dilated, tortuous, elongated veins along the medial right thigh, which are especially pronounced upon standing. The remainder of the physical exam is normal. What will be the most appropriate therapeutic approach for this patient at this time? a. furosemide b. sclerotherapy c. warfarin (Coumadin) d. clopidogrel (Plavix) e. cilostazol (Pletal)

b- sclerotherapy treats varicose veins & involves the injection of an irritating solution into the varicose vein to promote an inflammatory response, scarring, & obliteration of the lumen a- diuretics are ineffective for varicose veins c- warfarin is indicated for MI or CVA prevention & in patients with A-fib, mechanical heart valves, or DVT d & e- Cilostazol and clopidogrel both inhibit platelets. Clopidogrel is used in CAD and CVA prevention, as well as post-ST-segment MI. Cilostazol lowers lipid levels and is indicated in peripheral arterial disease.

An 87 yo F presents with recent onset of intractable headaches, jaw claudication, and visual field changes, including diplopia. Her PMH is significant for Polymyalgia Rheumatica. Based on the most likely diagnosis, what is the primary reason for prompt diagnosis and treatment of this condition? a. prevent hemorrhage b. prevent MI c. prevent blindness d. prevent stroke e. prevent facial palsy

c- Based on the classic presentation of jaw claudication, intractable headache, and visual field changes, your index of suspicion should be high for Giant Cell Arteritis. Prompt treatment is necessary to prevent permanent blindness; it can be caused by an occlusive arteritis of the ophthalmic artery. Treatment consists of prednisone, 60mg given daily, immediately and a temporal artery biopsy performed after treatment initiated. The other choices are not the primary reason for prompt diagnosis and treatment of giant cell arteritis; however, the others may also be prevented with prompt treatment.

A 57 yo M with a Hx significant for alcoholism presents with a 3-week history of dyspnea and fatigue. He has not seen a medical provider in multiple years and has no known medical diagnoses. He takes no daily medications. Work up reveals a normal troponin, elevated BNP, lateral T wave inversion on EKG, and evidence of cardiomegaly on CXR. You order a stat echo, which shows enlarged left ventricle and systolic heart failure evidenced by decreased contractility. What is the most likely cause of the patient's dyspnea and fatigue? a. COPD b. cardiac ischemia c. dilated cardiomyopathy d. acute MI e. acute pericardiitis

c- Dilated cardiomyopathy is caused by diminished cardiac contractility secondary to myocyte destruction.1 Multiple conditions can result in the death of the myocytes including chronic alcohol abuse and ischemia.1 The findings of an elevated BNP, cardiomegaly on CXR with decreased contractility, left ventricular enlargement, and systolic dysfunction on echo provide the diagnosis of dilated cardiomyopathy. a- Chronic obstructive pulmonary disease (COPD) may present with dyspnea and fatigue, but the patient will not have an elevated BNP and an abnormal echo. b- The patient's EKG does show evidence of ischemia, demonstrated by T-wave inversion, which was likely a causative factor for the development of dilated cardiomyopathy, but cardiac ischemia alone is not the cause of the patient's dyspnea and fatigue. d- The patient is not having an acute myocardial infarction, as evidenced by a normal troponin and no EKG evidence of infarction. e- Acute pericarditis presents pleuritic chest pain and work-up reveals global ST elevations on EKG with normal laboratory studies and a normal chest X-ray.

A 30 yo African American man develops dysuria and increased frequency of micturition. In the ED, he is found to have a UTI and is treated with trimethoprim-sulfamethoxazole (Bactrim), as well as recommended to follow up with his primary care physician in the office. Over the next few days, the patient experiences fatigue, fever, jaundice, abdominal and back pain, and dark urine. Blood tests show Hb of 4 g/dl, reticulocyte count of 6%, and MCV of 93. Peripheral smear reveals cell fragments, microspherocytes, and blister or bite cells. Heinz bodies are also present. Based on these findings, which clinical entity is most likely? a. sickle cell anemia b. hereditary spherocytosis c. G6PD deficiency d. autoimmune hemolytic anemia e. porphyria

c- G6PD deficiency is a very common X-linked hemolytic disorder affecting millions across the world. The highest incidence is among people of African, Asian, or Mediterranean descent. Glucose-6-phophate dehydrogenase is an enzyme in red cells that is vital to the integrity of the cell. Deficient individuals are susceptible to hemolysis under oxidant stress caused by drugs like sulfa and antimalarials, infections, or noxious agents like fava beans. Oxidant stress causes denatured hemoglobin, resulting in Heinz bodies. During an acute event, as in this patient, rapid hemolysis causes acute anemia, hemoglobinuria (thus the dark urine), abdominal and back pain, jaundice, and even renal failure. G6PD levels are normal to low during an acute attack, since the replacement reticulocytes are young and have adequate enzyme levels. G6PD level should be measured after an acute attack is over. Removing the precipitating agent and getting oxygen and rest is usually sufficient during an acute attack. a- Sickle cell anemia is a hemoglobin disorder mostly occurring in African Americans in which there is a mutation in the amino acid sequence in the globin chains, resulting in severe structural and functional changes in the red cells. The hemoglobin molecule gets polymerized, resulting in rigidity of the molecule and causing red cells to have an altered shape resembling a sickle. This predisposes the red cells to undergo hemolysis with stressors like infections. Patient may have crises of different types, including aplastic, painful, hemolytic, etc. Gallstone disease may occur. Frequent visits to the ER are common with chest pain (acute chest syndrome), abdominal pain from splenic infarctions, and infections due to encapsulated organisms. Acute chest syndrome is defined as a new infiltrate on chest X-ray along with new symptoms such as cough, tachypnea, dyspnea, and fever. It is probably due to pulmonary infarction, due to sickle cells along with infection. Chest X-ray, sputum cultures, and blood cultures are re b- Hereditary spherocytosis is another type of hemolytic anemia due to congenital red cell membrane defect, causing red cell swelling and resulting in the typical small round spherocytes without any central pallor, as in a normal RBC. This results in increased osmotic fragility, and red cells are easily destroyed in the spleen. Symptoms vary from incidental diagnosis to severe anemia, to neonatal hyperbilirubinemia, to hydrops fetalis in utero. Patients may present with biliary symptoms in adulthood. d- Autoimmune hemolytic anemia occurs due to development of antibodies directed at red cells, resulting in their destruction. It occurs in patients with systemic autoimmune disorders like SLE and ulcerative colitis, lymphoproliferative disorders like lymphomas, and in AIDS. Steroids and splenectomy may be needed to treat this condition. Patients present with anemia, jaundice fever, abdominal pain, and dark urine. Labs show anemia with normal RBC indices, increased reticulocyte count, nucleated red cells and spherocytes in the peripheral smear, and erythroid hyperplasia in the bone marrow. Coomb's test may be positive. Transfusion may be life saving. e- Porphyria is an autosomal dominant disease due to genetic or acquired deficiencies in the activity of enzymes in the heme biosynthetic pathway. Symptoms include abdominal pain, dark urine, and neurological and psychiatric manifestations. UA shows high porphyrin content.

During an annual physical exam, your patient states that although she does not currently have an outbreak, she has had 7 outbreaks of genital herpes this year. She asks you how she can prevent future outbreaks. As per CDC recommendations, what action will you take? a. apply abreva (Docosanol) topical ointment to the vesicles b. teach her how to douche c. place her on suppressive therapy d. encourage her to avoid sexual contact during outbreaks e. have the patient use a sitz bath to relieve the pain of the vesicles

c- If patient is having a recurrent herpes outbreak (> 6 outbreaks a year), placing a patient on suppressive therapy decreases recurrence by 70 - 80%. Douching changes vaginal flora and may lead to vaginitis. Douching does not treat HSV. Although avoiding sexual contact during outbreaks is recommended, this does not prevent outbreaks. Sitz bath is not a method used to prevent outbreaks. Abreva (Docosanol) is used to treat acute herpes labialis, not genital herpes.

A 65 yo Hispanic-American man presents with a 3-month history of chronic intermittent diarrhea with 5-6 watery stools per day; some are tinged with blood. During this period, he has had progressive fatigue and intolerance to exercise; he has lost 15 pounds (6% of body weight). Vital signs are within normal ranges. On physical examination, there is noticeable pallor. Heart sounds are regular and of normal frequency, both lung fields are clear to auscultation, the abdomen is soft and diffusely tender to palpation, particularly in the right quadrants. Rectal examination is painless, and an enlarged prostate is palpated, but there are no masses or blood. Anoscopy shows grade I hemorrhoids. There is no peripheral edema. A CBC shows the following: Ht 30% Hb 9.4 g/dL MCV 77 fl MCHC: 25 g/dL WBC 8,600 /mm3 normal differential platelets 460,000 /mm3 iron 56 µg/dL ferritin 5.5 µg/dL What is the most appropriate next step in management? a. sigmoidoscopy b. CT abdomen c. full colonoscopy d. fecal occult blood test e. barium enema

c- Iron-deficiency anemia in an adult should prompt for evaluation of a gastrointestinal source of bleeding. This is especially important because a malignant etiology, particularly colorectal cancer, is possible. The signs and symptoms of colon cancer vary according to the tumor's site in the colon. Distal tumors are more frequently associated with hematochezia and obstruction, while proximal neoplasia tends to produce more chronic obstruction with proximal dilatation, intermittent bleeding with iron-deficiency anemia, and a palpable mass. Recurrent infarctions lead to intermittent luminal obstruction and episodic diarrhea. Some lesions grow to a considerable size before diagnosis. In this patient, the diagnostic technique with the highest yield is full colonoscopy. It permits visualization of the whole colon; it also permits performing a biopsy for histopathologic confirmation of the diagnosis. In selected cases, colonoscopic dilatation or stent placement can alleviate obstruction before surgery is performed. Some authorities recommend routine upper endoscopy while investigating lower gastrointestinal bleeding. In some series, upper endoscopy revealed a missed bleeding source in as many as 10% of patients, and altered management in nearly 50% cases. d- A fecal occult blood test (FOBT) is redundant, given the presence of active bleeding and the overt signs of iron-deficiency anemia. FOBT is useful in colon cancer screening, despite its high false-positive rates (e.g., from non-cancer bleeding or foods reactive with the test reagent). Another caveat is that cancers may bleed intermittently, thereby yielding a false-negative result on a random FOBT. e- Barium enema is less sensitive than colonoscopy and is unable to perform biopsy. Despite these disadvantages, some authorities recommend its use for colon cancer screening.

A 27 yo M presents with increased urination and thirst. Over the past 3 days, he has been unable to satisfy his thirst and has to urinate up to 20 times per day. He noticed that his urine is very clear and colorless. In general, he feels very weak. He has never had any problems with urination before. His medical history is remarkable for a recently diagnosed psychiatric condition for which he began medical treatment. On physical exam, he appears to be lethargic, dehydrated, and pale. His vital signs are as follows: blood pressure 96/52 mm of Hg, temperature 101.2 degrees Fahrenheit, pulse 108 beats per minute, and a respiratory rate of 26 per minute. Which of the following is a common cause of diabetes insipidus? a. vasopressin b. valproic acid c. lithium d. amitriptyline e. carbamazepine

c- Lithium, used in the treatment of bipolar disorder, is a common cause of diabetes insipidus. It is freely filtered through the glomerulus and reabsorbed in the proximal tubule along with sodium and water. Even small doses of lithium may cause diabetes insipidus. Lithium can also make the distal renal tubules resistant to the action of vasopressin. This patient had recently started this medication, which resulted in his new symptoms b, d, e- Carbamazepine, amitriptyline, and valproic acid do not cause diabetes insipidus. a- Analogues of vasopressin are used to treat diabetes insipidus.

A 39 yo M presents with atrial flutter with 2:1 AV conduction, giving him a pulse of 150 per minute, which is perfectly regular. He takes no medications regularly. You plan to provide him with urgent direct current cardioversion with conscious sedation. What should be used as the initial energy for cardioversion in order to restore sinus rhythm in patients with atrial flutter? a. 300 Joules b. 200 Joules c. 100 Joules d. 25 Joules 3. 10 Joules

c- Of all of the arrhythmias, both supraventricular and ventricular, atrial flutter is the easiest to cardiovert back to a regular sinus rhythm. The temptation, especially when using front-to-back paddles, is to use the lowest voltage setting (e.g., 50 Joules, or 25 Joules, or even 10 Joules). That temptation needs to be resisted; if the initial electric shock is ineffective, then the circulating catecholamine levels of the patient become markedly increased, which thereby places the patient at risk of developing potentially fatal ventricular fibrillation and cardiac arrest with subsequent electric shocks. In contrast, there is no need to apply higher energies such as 200 Joules, 300 Joules, or 360 Joules as the initial energy for cardioversion in case of atrial flutter. The most reasonable approach is to provide a 100 Joules direct current countershock which will have a high probability of returning the patient with atrial flutter to a regular sinus rhythm while maintaining a low probability of causing burns or broken bones with 1 and only 1 direct current moderate-energy 100 Joules electric shock.

A 31 yo HIV positive man presents to your clinic for the first time with a history of pneumocystosis, and he is on trimethoprim/sulfamethoxazole for secondary prophylaxis. Primary prophylaxis of Pneumocystis pneumonia is recommended for patients at what CD4 cell count? a. 500 cells/mcL b. 350 cells/mcL c. 200 cells/mcL d. 100 cells/mcL e. 50 cells/mcL

c- Primary prophylaxis for pneumocystosis in a patient with HIV infection should be initiated when CD4 cells counts drop to 200 cells/mcL or less. When CD 4 levels drop to 50 cells/mcL, primary prophylaxis for mycobacterium avium complex should be initiated. The other cell counts listed are not levels at which primary prophylaxis should be initiated. e- When CD 4 levels drop to 50 cells/mcL, primary prophylaxis for mycobacterium avium complex should be initiated. The other cell counts listed are not levels at which primary prophylaxis should be initiated.

What is the therapeutic window for using recombinant tissue plasminogen activator in acute ischemic stroke? a. 6 hours b. 4 hours c. 3 hours d. 2 hours e. 1 hour

c- TPA catalyzes the conversion of plasminogen to plasmin, which promotes fibrinolysis. Ischemic stroke is caused by sudden occlusion of a cerebral artery by thrombus or embolus, producing a focal neurologic deficit. Timely reperfusion of ischemic brain tissue has been shown to limit irreversible neuronal injury in both humans and animals. The benefit of TPA in acute ischemic stroke was demonstrated in a large, multicentered national study. Patients whose symptoms began within 3 hours of treatment with TPA had significantly less disability at the end of 3 months than those given placebo. Although treated patients had higher incidence of secondary brain hemorrhage (6.4% vs. 0.8 %), there was no difference in mortality between the 2 groups. TPA must be given no later than 3 hours of symptom onset in patients with acute ischemic stroke. Prior to treatment, it is essential to exclude cerebral hemorrhage on a head CT scan. The presence of a bleeding diathesis, uncontrolled hypertension, recent prior stroke, and past history of cerebral hemorrhage are important contraindications for using TPA in acute ischemic stroke. TPA given later than 6 hours of stroke onset is associated with unacceptable risk of cerebral hemorrhage; it is not known whether fibrinolytic therapy can be given between 3 and 6 hours of stroke onset.

A 14 yo F presents with a 1-hour history of rapid heartbeat, faintness, sweating, and nervousness. She is also experiencing shortness of breath and chest pain. The patient has no significant past medical history. There is no history of similar episodes. The patient is on no medications & denies illicit drug use. On exam, her vital signs are BP 70/60 mmHg; pulse 200 bpm; RR 22/min, temperature afebrile. She looks pale, and her palms are slightly sweaty. She is not comfortable sitting up, so she prefers lying down. She looks slightly apprehensive. Her heart and lung exam are negative except for the tachycardia; except for cool sweaty hands, a brief abdominal and extremity exam are non-revealing. The physician quickly places the paddles on the patient's chest to record the rhythm; this shows a narrow-complex regular tachycardia at 210 bpm. He requests oxygen, IV line, and continuous monitoring. An EKG is in the process of being completed. At this point, what should be done? a. diltiazem 10 mg IV push b. verapamil 5 mg IV push c. synchronized cardioversion d. carotid sinus massage e. adenosine 6 mg IV push

c- Tachycardia can be classified based on the appearance of the QRS complex on the ECG as narrow and wide complex tachycardia. Narrow complex tachycardia consists of sinus tachycardia, atrial fibrillation, atrial flutter, AV nodal reentry, and atrial tachycardia (ectopic and reentrant). Wide complex tachycardia consists of ventricular tachycardia and supraventricular tachycardia with aberrancy. The patient presents with a tachycardia and associated serious symptoms of faintness, shortness of breath, chest pain and apprehension, mild hypotension, and peripheral vasoconstriction. Thus, while young patients can tolerate a rapid heartbeat for some time, this patient would be classified as hemodynamically unstable. The treatment of choice for unstable patients with a narrow complex tachycardia would be immediate synchronized cardioversion. However, whenever possible, it is advisable to provide analgesia and sedation for conscious patients before cardioversion. a, b, & e- If the vagal maneuvers do not succeed, the physician may wish to give a fluid bolus and try medications such as adenosine (6mg rapid IV push over 3 seconds). Adenosine works in 90% of narrow complex tachycardias, and with vagal maneuvers, it can be viewed as the initial management choice. The adenosine can be repeated twice at doses of 12 mg IV; the doses must be spaced at 1 - 2 minutes apart, and the adenosine must be injected in a vein that is close to the heart (at the very least, a brachial vein with elevation of the arm); it should be followed by a 20cc saline push. If there is still no rate conversion, then diltiazem or verapamil can be administered. d- In stable patients, vagal maneuvers such as carotid sinus massage or Valsalva maneuver should be the initial approach; it should be followed by administration of adenosine. Performing a carotid sinus massage and asking the patient to perform a Valsalva maneuver are relatively innocuous means of increasing vagal tone, and there is the potential for breaking the narrow-complex tachycardia.

A 46 yo M presents with a 24-hour history of severe anal pain and swelling. The pain started after straining at defecation and has worsened over the course of the day. There is no history of fever. Examination of the anal area reveals a swollen ecchymotic mass in the perianal skin, very close to the anal verge. What is the treatment of choice for this condition? a. antibiotics b. immediate surgical drainage in the OR c. excision of thrombosed external hemorrhoid d. internal sphincterotomy e. immediate colonoscopy

c- The clinical case described most likely represents a patient with a thrombosed external hemorrhoid. External hemorrhoidal thrombosis is not infrequent and can be seen in patients with no prior history of hemorrhoidal disease. The cause is thought to be from elevated venous pressures related to excess straining with constipation and physical exertion following a bout of diarrhea or change in diet. It is an acute painful event with the pain generally lasting 7-14 days and resolving with resolution of the thrombosis. The clinical presentation is usually as described above, with acute swelling and pain located at or around the anal verge. When the overlying skin of the thrombosed hemorrhoid is eroded, bleeding occurs. The pain usually lasts for a few days and then spontaneously resolves. The swelling may take several weeks to resolve. Treatment depends on the time of presentation and the severity of the condition. Acute thrombosed external hemorrhoid presenting within 72 hours of onset of symptoms should be excised. Excision of the thrombosed external hemorrhoid can be performed under local anesthesia and is associated with a low recurrence and complication rate. Patients presenting after 72 hours of onset of symptoms should be managed by conservative/symptomatic medical therapy.

A 57 yo M presents with moderately severe chest pain. His pain is substernal and left anterolateral; there is some exacerbation on inspiration, and it has been increasing in severity over the last 36 hours. He works as a truck driver and has a history of heavy cigarette smoking, hypertension, and obesity. Over the past week, he has experienced swelling and discomfort in his right calf. Examination shows: BP - 90/55 mm Hg, P - 122/min, RR - 40/min, and temp - 37.6° C. He is mildly agitated and confused. Systemic examination reveals: heart: tachycardia, soft systolic murmur, questionable ventricular gallop. Chest: dullness to percussion left base with scattered crackles and wheezes throughout. Abdomen: negative. Extremities: right calf is 0.5 cm larger than left with some deep tenderness and a trace of ankle edema. Neurologic: no deficits. Laboratory: Hg- 16.4g/dL, Hct 51%, WBC 12,300 cells/ µL, PaO2 52 mmHg, PaCO2 38 mmHg, and pH 7.35. Chest radiograph: borderline cardiomegaly and a prominent aorta; scattered patchy infiltrates bilaterally, small left pleural effusion. What is the most accurate diagnostic modality for diagnosing this patient's condition? a. spirometry b. impedance plethysmography c. CTA d. myocardial scan e. cardiac ultrasound

c- The clinical picture is most compatible with a diagnosis of pulmonary embolism (PE). Pulmonary embolism is a condition in which there is sudden lodgment of a blood clot in a pulmonary artery with cause subsequent obstruction of blood supply to the lung parenchyma. The clinical manifestations of PE are usually nonspecific and vary in frequency and intensity depending on the extent of pulmonary vascular occlusion, pre-embolic cardiopulmonary function, and the development of pulmonary infarction. Small thromboemboli may be asymptomatic. The most common symptoms of PE include dyspnea, pleuritic chest pain, cough, and hemoptysis. Based on the acuity and severity of pulmonary arterial occlusion, PE can be categorized into 4 types as massive PE, acute pulmonary infarction, acute embolism without infarction, and multiple pulmonary emboli. CTA is the initial imaging technique of choice in stable patients. The ventilation perfusion scanning (V/Q)/lung scan is also an important diagnostic modality for the diagnosis of PE with a reasonable possibility of either confirming the diagnosis (with a "high-probability" scan) or ruling it out (with a normal scan). However, CTA has largely replaced it as the initial modality of imaging in stable patients and V/Q scanning is usually done if CT is not available or the person has a contraindication to CT or the intravenous dye. a, d, e- These tests address other possible diagnoses that are either of secondary importance (spirometry) or less likely (myocardial scan and cardiac ultrasound) b- Impedance plethysmography could be helpful in explaining the leg tenderness and identifying the source of emboli, but it would not be definitive in confirming pulmonary embolism.

A 57 yo F comes to your clinic presenting with episodic vertigo, tinnitus, hearing loss, and ear fullness. Her ear and eye physical examination are unremarkable. You perform a Dix-Hallpike maneuver which is negative. There are no carotid bruits noted on auscultation. Which of the following is the best initial treatment for this patient? a. diazepam 10 mg BID b. restrict water intake c. low sodium diet d. antibiotics e. fluticasone propionate

c- The clinical picture is suggestive of Meniere's disease. The classic syndrome consists of episodic vertigo lasting 20 minutes to several hours associated with fluctuating low-frequency sensorineural hearing loss, tinnitus, and a sensation of aural pressure. The Dix-Hallpike maneuver is a diagnostic maneuver for benign paroxysmal positional vertigo. Treatment involves a low sodium diet and diuretics. a- Diazepam can be used for Meniere's disease but is usually used for severe vertigo. The question is indicating initial treatment. Diazepam is used in the treatment of vestibular neuronitis. In vestibular neuronitis, a paroxymal, usually single attack of vertigo occurs without accompanying impairment of auditory functions and will persist for several days to weeks before clearing. Examination reveals nystagmus. These symptoms are not present in this patient. b- Restricting water intake will lead to dehydration and an increase in sodium levels, worsening the symptoms of Meniere's disease. d- Antibiotics are not indicated for Meniere's disease. e- Fluticasone propionate is an anti-inflammatory nasal spray used to treat the nasal symptoms of indoor and outdoor nasal allergies and year-round nonallergic nasal symptoms. Fluticasone helps reduce the inflammation that leads to nasal symptoms that include congestion, sneezing, and itchy, runny nose which are not indicated in this patient.

A 38 yo M presents with a 1-week history of severe chest pain. He states that the pain seems to worsen when he lies down. He describes the pain as radiating to the back, and it also worsens when he takes a deep breath. His vital signs are as follows: blood pressure 124/ 84 mm Hg, respiratory rate 18/ min, temperature 101°F, and pulse 74 beats per minute. On auscultation of the chest, you cannot distinguish a S1 or S2 but hear a scratching or grating sound. What is the first step in the treatment of this patient? a. corticosteroids b. no treatment necessary c. rest & NSAIDs d. pericardiocentesis e. beta blockers

c- The clinical picture is suggestive of acute pericarditis. Most cases are due to viral infections with the treatment being rest and non-steroidal agents, e.g. aspirin or indomethacin. If this pericarditis progressed to tamponade, pericardiocentesis would be indicated. Symptoms of tamponade are not seen in this patient (dyspnea, elevated jugular venous pressure, hypotension, paradoxic pulse, and muffled heart sounds). Beta blockers are not indicated for treating pericarditis. Corticosteroids are usually given in cases unresponsive to rest and NSAIDs. Most cases of pericarditis are self-limiting and usually run their course from 1-3 weeks but initial treatment consists of rest and NSAIDs. a- Corticosteroids are usually given in cases unresponsive to rest and NSAIDs. b- Most cases of pericarditis are self-limiting and usually run their course from 1-3 weeks but initial treatment consists of rest and NSAIDs. d- If this pericarditis progressed to tamponade, pericardiocentesis would be indicated. Symptoms of tamponade are not seen in this patient (dyspnea, elevated jugular venous pressure, hypotension, paradoxic pulse, and muffled heart sounds). e- Beta blockers are not indicated for treating pericarditis.

A family presents in the middle of winter. They live in a low-income housing development. Their gas furnace is broken, and they have been using a kerosene heater at night. For the last 3 days, they have all been experiencing varying degrees of headache, dizziness, nausea, vomiting, and fatigue; symptoms are particularly severe at night. Their 4 yo child has also been very lethargic; occasionally, she seems to black out or fall asleep very soundly. She and her 9 yo sibling have also had a cough, runny nose and sore throat for the past week. On exam, other than seeming tired, findings are nonspecific. Both children have a runny nose but their lungs and ears are clear. What is the best test to confirm exposure of the most likely diagnosis in this case? a. hemoglobin level b. urinalysis c. carboxyhemoglobin level d. blood gas analysis e. pulse oximetry

c- The family has CO poisoning. Carboxyhemoglobin is formed when inhaled CO binds to hemoglobin after being absorbed into the bloodstream. CO has an affinity for hemoglobin that is 250 times that of oxygen. Both oxygen transport and delivery to tissues are reduced as carboxyhemoglobin interferes with the dissociation of oxygen that should be provided until symptoms resolve and carboxyhemoglobin levels decrease to 5% or less. Delivery of 100% oxygen reduces the elimination half-life of CO to 1 hour from 4-5 hours. a- Hemoglobin level gives no indication of how much is bound to oxygen versus how much may be bound to CO. Thus, a hemoglobin level may be normal, and yet most of it may be bound to CO as carboxyhemoglobin b- Urinalysis may show myoglobinuria due to eventual muscle necrosis as CO binds to myoglobin, decreasing its oxygen-carrying capacity. d- Blood gas analysis may show metabolic acidosis with a normal arterial oxygen tension (PaO2) as measurement of dissolved oxygen, and it can overestimate the true oxygen saturation of hemoglobin. e- Pulse oximetry that measures oxygen saturation is falsely normal because oxyhemoglobin and carboxyhemoglobin cannot be differentiated. Pulse oximeters use 2 light emitting diodes (a red one and an infrared light). Oxygenated hemoglobin absorbs infrared light and nonoxygenated hemoglobin absorbs red light. The pulse oximeter determines the relative absorption of each and gives the percentage of oxygenated versus nonoxygenated hemoglobin present. With significant carboxyhemoglobinemia, the oximeter will reflect only the oxygen saturation of normal hemoglobin and not the percentage of hemoglobin bound to carbon monoxide.

An 83 yo M was treated for ventricular arrhythmias. He presents 1 month later with joint pains. He also has an unusual mask-like rash over his face and body. Discontinuation of drug therapy causes the symptoms to abate. What drug was most likely administered to this patient? a. propranolol b. phenytoin c. procainamide d. quinidine e. tocainide

c- This question addresses issues related to the treatment of arrhythmias and lupus-like side effects. The initial treatment should control arrhythmias using lidocaine, unload the heart using a balanced vasodilator such as nitroglycerin or nitroprusside, and use aspirin and heparin to prevent further clot formation. Once discharged, patients are frequently placed on atenolol to control catecholamine-induced arrhythmias. Finally, long-term control of arrhythmias is accomplished using drugs such as procainamide or quinidine. Unfortunately, procainamide can cause lupus-like side effects. The use of thrombolytics has reduced myocardial damage caused by thrombus formation. Streptokinase, urokinase, and tissue plasminogen activator (tPA) all will lyse clots. This leads to coronary reperfusion. If performed within the first several hours post-clot, myocardium will be spared. The use of aspirin and heparin are directed at prevention of platelet aggregation and clot stabilization. a- Propranolol is a beta-blocker and does not cause lupus-like side effects. b- Phenytoin is an anticonvulsant not an antiarrhythmic but may cause lupus-like side effects in a small number of people. d & e- Tocainide and quinidine are antiarrhythmics but do not cause lupus-like side effects.

A 17 yo white F presents with a 6-month history of blackheads and whiteheads on her face. On examination, you find a few papules and pustules on her cheeks; there are no nodules. Her mother reports that she had similar spots on her face before she got married. What is an appropriate first-line medication for her? a. intralesional triamcinolone b. oral minocycline c. topical tretinoin d. oral doxycycline e. oral isotretinoin

c- Topical retinoids, such as tretinoin, adapalene, and tazarotene, are effective first-line treatments for mild acne. Tretinoin is a comedolytic, which inhibits follicular canal obstruction by normalizing keratinocyte shedding, thereby inhibiting microcomedone formation. It also has anti-inflammatory properties. Adapalene has anti-inflammatory effects and normalizes follicular cell differentiation. The clinical presentation is that of acne vulgaris. Acne is a common skin condition that affects the pilosebaceous units. It usually begins in adolescence, though it also affects adults, especially women. Acne can be classified as mild, moderate, or severe. In mild acne, there are comedones, a few papules, and pustules without any nodules. In cases of moderate acne, patients have comedones, several papules and pustules, and a few nodules. In cases of severe acne, there are numerous comedones, papules, pustules, and nodules. Other topical agents such as benzoyl peroxide, topical sulfacetamide, and topical azelaic acid, are also effective first-line treatments for mild acne. Topical antibiotics, like erythromycin and clindamycin, are used in mild-to-moderate inflammatory acne because they have anti-inflammatory properties in addition to being effective against P. acnes. Benzoyl peroxide and topical antibiotics combinations reduce antibiotic resistance. a- Intralesional corticosteroid injections reduce the inflammation in nodulocystic acne. Triamcinolone is injected into the cyst, and care should be taken because it can cause atrophy and depressed scars. b & d- Moderate to severe inflammatory acne requires systemic antibiotics such as oral tetracycline, doxycycline, minocycline, and erythromycin. They have anti-inflammatory effects in addition to battling the P. acnes colonization. They can increase resistance to erythromycin and cause possible GI disturbances; tetracycline should be avoided during pregnancy and in children under 9 years. e- You chose: Topical tretinoin The correct answer is: Topical tretinoin Topical retinoids, such as tretinoin, adapalene, and tazarotene, are effective first-line treatments for mild acne. Tretinoin is a comedolytic, which inhibits follicular canal obstruction by normalizing keratinocyte shedding, thereby inhibiting microcomedone formation. It also has anti-inflammatory properties. Adapalene has anti-inflammatory effects and normalizes follicular cell differentiation. The clinical presentation is that of acne vulgaris. Acne is a common skin condition that affects the pilosebaceous units. It usually begins in adolescence, though it also affects adults, especially women. Acne can be classified as mild, moderate, or severe. In mild acne, there are comedones, a few papules, and pustules without any nodules. In cases of moderate acne, patients have comedones, several papules and pustules, and a few nodules. In cases of severe acne, there are numerous comedones, papules, pustules, and nodules. Other topic

A 55 yo M presents with a 3-day history of severe pain associated with bowel movements. The pain lasts up to 1 hour following each bowel movement. His stools are described as "hard", but they retain their normal brown color, with the occasional presence of a few drops of bright red blood. His dietary history is remarkable for a low-fiber diet lacking in raw fruits and vegetables. He denies any sexual contact, drug use, or other gastrointestinal symptoms. He takes ferrous sulfate for anemia and oxycodone for chronic joint pain. His rectal exam reveals a small tear in the lining of the anus. What is the most appropriate diagnostic test to order at this time? a. rectal biopsy b. anoscopy c. no diagnostic tests are necessary d. HIV ELISA test e. CT scan of the abdomen & pelvis

c-This patient's diagnosis is a posterior anal fissure. Along with a history, the diagnosis can usually be made based on findings from a gentle perianal examination with inspection of the anal mucosa. In this case, no diagnostic procedures are required. A digital rectal examination is painful and often can be deferred. Anoscopy may be required for lesions that are not well-visualized; however, this is not well tolerated by a patient with an acute anal fissure. In this case, anoscopy can often be deferred; the patient can be treated based only on the symptoms. If an ordinary anal fissure is suspected, and if it is located in the posterior or anterior midline, then no laboratory or imaging tests are necessary. If the fissure is off the midline or irregular, or if an underlying illness (e.g., Crohn disease, squamous cell cancer, AIDS) may be present, then the appropriate tests should be ordered. These tests include erythrocyte sedimentation rate, stool and viral cultures, human immunodeficiency virus (HIV)testing,and biopsy of the lesion/fissure.

The parents of a 3 yo boy asks to have a blood test done on their son for lead poisoning. He has not been tested before. They have moved into an older home built before 1960. They has noticed some peeling paint on windowsills and doors and has seen small paint chips on the floors. They are now having the house repainted and are staying with relatives. A careful environmental history is obtained, risk reduction and nutrition education is provided. His fingerstick blood lead level comes back at 13mcg/dL. Which additional management should be done at this level? a. Consider retesting within 3 months b. Obtain abdominal radiographs and order bowel decontamination if indicated c. Collaborate with lead poisoning preventive program to provide home inspection and other services d. Repeat capillary samples, confirming with venous sample within 1 month e. Repeat with a venous sample within 1-3 months

d- A blood lead level of 10-14 mcg/dL is considered moderate risk II and warrants, in addition to reporting to the local health department, repeating all capillary samples and confirming with a venous sample within 1 month for new cases and 1-3 months for known cases. As this is a new case, venous testing should be done within 1 month after repeating capillary sample. Lead based paint in older housing currently is the most important source of lead exposure. Young children's mouthing behavior and pica are widely recognized as causes of elevated blood lead concentrations. Lead is toxic to most body systems, including the nervous system, liver, and hematopoietic and renal systems. Lead irreversibly binds and impairs function of enzymes in the pathway to heme synthesis. Blood levels at which lead poisoning is defined and corresponding interventions has changed several times over the past decades. Improved understanding of subclinical toxicity on neurodevelopment has led to proactive screening and environmental intervention to prevent elevated lead levels. Treatment of lead poisoning varies with the blood lead level, though all management should focus on nutritional and environmental intervention. Chelation therapy is the mainstay of treatment for lead levels greater than 45 mcg/dL. a- Retesting in 3 months would be a safe course of action when dealing with blood lead levels less than 10 mcg/dL and risk assessment indicates that lead exposure is likely. b- Abdominal radiographs and bowel decontamination are indicated at lead levels of 20-44 mcg/dL if particulate ingestion is suspected. c- Collaboration with lead poisoning prevention programs that will provide home inspection and other services should be done at blood lead levels of 15-19 mcg/dL and lead levels followed. e- In known cases of lead levels between 10-14 mcg/dL, a confirmatory venous sample can be done within 1-3 months; however, as this is a new case, testing should be done within 1 month.

A 47 yo F with a 25-year history of T1DM presents for follow-up of her diabetes. Her spot albumin/creatinine ratio was 100 mg/g 4 months ago, and it was confirmed at 100 mg/g yesterday. Her urinary analysis shows no cells, casts, or blood. Her creatinine is 0.7 mg/dl, and her estimated glomerular filtration rate is 95 ml/min/1.73 m2. How would her proteinuria be described? a. normoalbuminuria b. minialbuminuria c. macroalbuminuria d. microalbuminuria e. nephrotic syndrome

d- Screening for microalbuminuria, a marker of kidney disease, is recommended annually for patients with type I diabetes of greater than 5 years duration; it is recommended annually for all patients with type II diabetes. This screening test is most easily obtained with a first void morning urine "spot" albumin to creatinine ratio. Normal levels for spot samples are <10 mg/g for men and <15 mg/g for women. Since kidney diseases other than diabetic kidney disease may cause proteinuria and albuminuria, evaluating the urine sediment of diabetics with presumed diabetic kidney disease is advised. Minialbuminuria is not a true medical term. For true medical terms used to describe increasing levels of albumin excretion, please see the table.

A 56 yo African American man presents as febrile with massive swelling of the abdomen and diarrhea. He has a 20-year history of heavy alcohol use. A fluid wave is elicited on physical examination of the abdomen by striking one flank and feeling the transmitted wave on the opposite flank. In what case would ascitic fluid analysis suggest cirrhosis as a cause of ascites in this patient? a. if fluid is designated as an exudate b. if the acidic fluid & leukocyte count is more than 500/mm3 c. if the serum ascites albumin gradient is lower than 1.1 g/dL d. if the fluid has a protein concentration below 3 g/dL e. if there is a large number of red blood cells in the fluid

d- After the diagnosis of ascites is made by physical examination, all patients with new-onset ascites should undergo abdominal paracentesis and ascitic fluid analysis. The most important tests to order for fluid analysis include protein concentration and cell count. Fluids with protein concentration above 3 g/dL are designated as exudates. Those with values below 3 g/dL are designated as transudates. Diseases usually associated with transudates include congestive heart failure, cirrhosis, constrictive pericarditis, inferior vena cava obstruction, hypoalbuminemia, Meigs syndrome, and some cases of nephrotic syndrome. The amount of albumin in the ascitic fluid compared to the serum albumin (the Serum Ascites Albumin Gradient, SAAG) can be indicative of the cause of ascites. Ascites related to hypertension, cirrhosis, or congestive heart failure generally shows a SAAG greater than 1.1 g/dL. Exudates are more commonly seen with peritoneal neoplasm, pancreatic ascites, myxedema, and tuberculous peritonitis. A large number of red blood cells in the fluid or grossly bloody ascites suggests a diagnosis of neoplasm. An acidic fluid and leukocyte count of more than 500/mm3 strongly suggests a peritoneal infection or inflammatory process. Other tests that should be ordered in the appropriate clinical setting include cytologic examination, lactic dehydrogenase (LDH), specific tumor markers, glucose, and cultures for bacteria, mycobacteria, and fungi. a- Exudates are more commonly seen with peritoneal neoplasm, pancreatic ascites, myxedema, and tuberculous peritonitis. b- An acidic fluid and leukocyte count of more than 500/mm3 strongly suggests a peritoneal infection or inflammatory process c- The amount of albumin in the ascitic fluid compared to the serum albumin (the Serum Ascites Albumin Gradient, SAAG) can be indicative of the cause of ascites. Ascites related to hypertension, cirrhosis, or congestive heart failure generally shows a SAAG greater than 1.1 g/dL. e- A large number of red blood cells in the fluid or grossly bloody ascites suggests a diagnosis of neoplasm.

The father of a 5 yo girl walks into her daughter's bedroom without knocking and discovers the girl stimulating her genitals. The girl's parents are concerned, but seem to be receptive to advice. What is the best response a physician could give? a. "This is not normal behavior for a child this age" b. "Don't you think you should knock before going into her room?" c. "Do you think that someone is molesting her?" d. "What disturbs you about this behavior?" e. "She probably has a vaginal infection. Bring her in so I can examine her."

d- Before the physician can provide guidance for the parents, the parents' concerns need to be understood. While the described behavior is perfectly normal for a 5-year-old, and it is appropriate for parents to knock on the door of their child's room before entering to teach children respect for privacy through modeling, the parents' concerns must first be understood. To immediately assume there is something physically wrong with the child, or that the child has been sexually abused, suggests that the physician may have some personal issues with children's normal sexuality.

A 51 yo M presents with a lump in his groin for 2 weeks. He states he was lifting an air conditioner and he felt a pop in his groin and began to notice an outpouching in his lower abdomen that has become mildly tender over the last week. Resting and lying flat appears to help, and standing and lifting aggravates it. He denies any fevers, nausea, vomiting, or changes in bowel habits. Patient denies any previous abdominal surgeries or procedures. Upon examination, you identify a soft, reducible mass in the lower abdomen and hernia examination reveals a mass pushing against the side of your finger. You order an ultrasound of the lower abdomen and find the intestinal sac has traversed through a weakened area of the abdominal wall and through Hesselbach's Triangle. This type of hernia is referred to as a(n)_____________. a. femoral hernia b. umbilical hernia c. incisional hernia d. direct inguinal hernia e. indirect inguinal hernia

d- Direct inguinal hernia is correct because the direct inguinal hernia enters through the weakened abdominal fascia and into the anatomic region known as Hesselbach's Triangle. This area is bordered by the rectus abdominus, the inferior epigastric artery, and the inguinal ligament. Femoral hernias, indirect inguinal hernias, and umbilical hernias do not traverse through Hesselbach's triangle. There was no previous surgery for an incisional hernia to occur.

A 25 yo F presents with an anaphylactic reaction after being stung by several bees. She c/o wheezing and SOB. On examination, the client is in acute distress. BP is 98/56 mmHg, PR 110/min, RR 28/min, and temperature 98.7°F. She is immediately treated with supplemental oxygen. In treating this condition further, what drug is required most urgently? a. methylprenisolone b. glucagon c. theophylline d. epinephrine e. cimetidine

d- Epinephrine is the drug of choice in anaphylactic reactions. The definite indications are hypotension, bronchospasm, and urticaria. Epinephrine has significant beta-agonistic effects, which cause bronchodilation, and alpha-agonistic effects, which can reverse systemic hypotension. IM route is preferred over SC, as time for maximum serum concentration is shorter by IM route. Patients with a previous history of similar insect bite or asthma must be given epinephrine. Side effects such as arrhythmias and angle-closure glaucoma can occur. Self-injectable preparations are available that can also be used to renew prescriptions. However, fluid replacement (especially when patient is in shock) and airway maintenance are also important. Diphenhydramine is an antihistaminic and H1 blocker that also must be used in all bee sting cases. It affords symptomatic relief from pruritus. a- Methyl prednisolone, a corticosteroid, is used in all generalized reactions for its anti-inflammatory property. b- Glucagon is required only if the patient is on beta-blockers. In such patients, it exerts its effects by inotropic and chronotropic properties, which are independent of beta-receptors. c- Theophylline may be required in resistant bronchospasm. e- Cimetidine is an H2 blocker, which may have an additive effect with H1 blockers in anaphylaxis.

A 40 yo F works full-time as a data entry clerk and often puts in many hours of overtime. She has started to notice numbness and tingling in her right thumb, index finger, middle finger, and half of her ring finger; symptoms are especially severe at night The numbness and tingling were intermittent for months, but they have become persistent during the past few days. What could be considered a predisposing factor for this patient's condition? a. Cushing syndrome b. hypoglycemia c. hypoadrenalism d. myxedema e. hypoprolactinemia

d- Myxedema is one of the metabolic predisposing causes for carpal tunnel syndrome. The patient's clinical presentation is suggestive of carpal tunnel syndrome. Carpal tunnel syndrome is a median nerve compressive neuropathy. Patients with myxedema or hypothyroidism have accumulation of myxedemateous tissue under the transverse carpal ligament, which causes compression of the median nerve in the carpal tunnel resulting in the manifestations of carpal tunnel syndrome. In carpal tunnel syndrome the median nerve is compressed in the wrist. The neurological distribution of her complaints corresponds to the distribution of the median nerve. The median nerve innervates the palmar surface of the thumb, index, middle, and radial half of the ring fingers and the motor branch innervates the thenar muscles. Electromyography and nerve conduction studies help in confirming the diagnosis of carpal tunnel syndrome. Patients can be managed with analgesics for pain, corticosteroids which can be administered orally or as local injection, and splint to prevent wrist flexion and further compression of the carpal tunnel. The other metabolic causes of carpal tunnel syndrome include acromegaly, diabetes mellitus, and amyloidosis. None of the other choices listed, which include hypoadrenalism, hypoprolactinemia, hypoglycemia, and Cushing syndrome, are associated with an increase in the occurrence of carpal tunnel syndrome.

A 33 yo M has a routine physical examination with no abnormal findings. His family history, however, is positive for familial adenomatous polyposis. What screening test would be best for him to obtain? a. AFP tumor marker b. stool DNA test (sDNA) c. fecal occult blood test d. colonoscopy e. no screening test necessary before the age of 50

d- The best test to recommend is a colonoscopy since it detects both polyps and cancer. Colorectal cancer is the 4th most common form of cancer in the United States; it arises from adenomatous polyps in the colon. Familial adenomatous polyposis (FAP) has different inheritance patterns and different genetic causes. In this patient, the pattern is probably autosomal dominant which puts him at a risk of nearly 100% for developing colorectal cancer by the age of 40. That means that screening tests for colorectal cancer must be performed earlier than suggested for the rest of the population (i.e., before the age of 50). A fecal occult blood test, fecal immunochemical test, and stool DNA test (sDNA) find cancer, not polyps. The AFP (alpha-fetoprotein) tumor marker is a useful marker for hepatocellular carcinoma and germ cell tumors, but not for colorectal cancer. a- The AFP tumor marker is a useful marker for hepatocellular carcinoma and germ cell tumors b & c- A fecal occult blood test, fecal immunochemical test, and stool DNA test (sDNA) find cancer, not polyps.

A 26 yo F presents to the ER after a syncopal episode. She had loss of consciousness 3 times over the past 12 months. Each event occurred during or just after physical exercise. On PE: BP 110/70 mm Hg, HR 75/min, normal S1/S2, and a III/VI systolic ejection murmur is heard best at the left sternal border that decreases with squatting. The EKG shows a normal sinus rhythm with diffuse increased QRS voltage. What is the most likely diagnosis? a. moderate mitral valve stenosis b. moderate pulmonary stenosis c. mild mitral valve insufficiency d. hypertrophic cardiomyopathy e. severe aortic stenosis

d- The clinical picture is suggestive of a Hypertrophic Cardiomyopathy (HCM). The age of presentation is usually in the mid-20's and the most frequent symptom is dyspnea although most patients are asymptomatic with a normal physical examination. Syncope may result from an arrhythmia due to abnormal myofibers. The transient increase in LV size during squatting reduces the LV outflow tract obstruction in HCM and softens the intensity of the murmur. The increase in QRS voltage indicates a hypertrophied heart and, depending on which leads (V1-V6), will tell you right or left heart. e- In Aortic stenosis, the murmur will get louder with squatting and in this patient the murmur gets softer. a- Mitral valve stenosis produces a diastolic murmur, while there is a systolic murmur in this patient. c- Mitral valve insufficiency (MVI) does produce a systolic murmur but it is described as pansystolic and the patient with MVI could have symptoms of shortness of breath, fatigue, palpitations, and orthopnea which is not present in this patient. b- Pulmonary stenosis is rare and would produce signs of jugular venous distention (JVD), cyanosis of nail bed, and general symptoms of hypoxia. There is no mention of JVD or cyanosis in this patient.

A 65 yo M presents to the office with polyuria and erectile dysfunction. He denies any other symptoms or significant past medical history. Physical examination reveals Tanner stage 5 of the external genitalia, balanitis of an uncircumcised penis, and slightly enlarged, symmetrical and smooth prostate. His condition is most likely the result of ____________. a. prostate cancer b. hypogonadism c. benign prostatic hypertrophy d. diabetes mellitus e. diabetes insipidus

d- The correct answer is diabetes mellitus since the presence of polyuria would indicate hyperglycemia and the associated erectile dysfunction and/or balanitis may be the only other presenting symptom or sign of diabetes mellitus in a male patient. Erectile dysfunction is a common vascular and neurological complication of diabetes and occurs in up to 75% of male diabetics. Elevated blood sugars result in autonomic neuropathy of the cavernous nerve of the penis so that erectile dysfunction serves as one of the earliest indications of neuropathy. Likewise, hyperglycemia results in microvascular damage to the dorsal and cavernous arteries, in the same way retinopathy, nephropathy, and neuropathy develop, further contributing to poor perfusion and erectile dysfunction. Hyperglycemia also results in the colonization of skin organisms, commonly Candida, resulting in typical superficial yeast infections seen in diabetics such as balanitis in men and vulvovaginitis in women. a- Prostate cancer most often develops in the peripheral zone of the prostate and is usually asymptomatic. Locally advanced prostate cancer may encroach on the central transition zone of the prostate and present with irritative urinary symptoms. Prostate cancer that extends outside the prostate capsule may result in erectile dysfunction. Carcinomas in the peripheral zone are often palpable and typically a hard, irregular nodule or induration. Prostate cancer is not typically associated with Candidaskin infections. b- Hypogonadism may present with fatigue, decreased libido, diminished erections, gynecomastia, or decreased testicular size, muscle mass, or hair growth associated with secondary sexual characteristics. It is typically not associated with an enlargement of the prostate, urinary complaints, or Candidaskin infections. c- Benign prostatic hypertrophy (BPH) typically occurs in the periurethral zone of the prostate and usually presents with lower urinary symptoms (LUTS) that suggest obstruction (i.e. hesitancy, weak stream, straining, post-void leaking) or irritation (i.e. nocturia, frequency, urgency). Digital rectal examination of prostatic hyperplasia typically reveals a smooth, firm enlargement of the gland which may be asymmetrical or indurated. Early BPH is not typically associated with erectile dysfunction or Candidaskin infections. e- The characteristic presentation of diabetes insipidus (DI) is abnormally large amounts of dilute urine - insipidus means tasteless. Polyuria is massive, often associated with nocturia and enuresis, and results in dehydration, which is often not evident due to a compensatory increase in thirst and polydipsia. DI is the result of the posterior pituitary's failure to secrete antidiuretic hormone (ADH) resulting in central diabetes insipidus (DI) or the kidney's resistance to ADH resulting in nephrogenic DI. DI is not typically associated with Candidaskin infections.

A 3 yo F presents with a 2-day history of sore throat and fever. This morning, she was hoarse and seemed to be having more difficulty breathing. On exam, she appears to be in distress and has an oral temperature of 100.0 F. Tympanic membranes are pink but not bulging. Nares are patent without rhinorrhea. She has a barking cough, stridor at rest, and nasal flaring. What treatment is most appropriate in the care of this child? a. start broad spectrum antibiotic b. discharge home to use cool mist vaporizer c. start specific antiviral therapy d. admit patient, start humidified oxygen & intermittent racemic epinephrine e- administer parainfluenza vaccine

d- This child is in acute respiratory distress as depicted by her stridor at rest, nasal flaring, and state of distress. The situation warrants admission, treatment, and careful observation. Corticosteroids via nebulizer may also be effective. Specific antiviral treatments and parainfluenza vaccines are not currently available. Parainfluenza virus will not respond to antibiotic treatment. Although home care with a vaporizer can be effective for patients with milder cases of croup, this child is severely ill. a- Parainfluenza virus will not respond to antibiotic treatment. b- Although home care with a vaporizer can be effective for patients with milder cases of croup, this child is severely ill. c & e- Specific antiviral treatments and parainfluenza vaccines are not currently available.

A 49 yo M presents with a 3-month history of weakness, fatigue, and abdominal discomfort. Upon further questioning, he acknowledges a lack of sexual desire. He denies any photosensitivity. On physical examination, his liver is enlarged, and his spleen is palpable. He has abnormal skin pigmentation on his face, neck and his elbows and which gives his skin a metallic gray hue. His laboratory results are in the chart. TEST RESULTS REFERENCE RANGE TIBC 275 μg/dL (250 - 350 μg/dL) Plasma iron 220 μg/dL (80 - 160) μ/dL Transferrin saturation 90% (16 - 57%) What serious complication is associated with the patient's condition? a. leukemia b. lymphoma c. bronchogenic carcinoma d. hepatocellular carcinoma e. pancreatic carcinoma

d- This patient has signs and symptoms c/w hemochromatosis, which is associated with HCC. Hemochromatosis is due to an increase in iron within the tissues. Hemochromatosis is a disorder of iron overload; it could be due to genetic or non-genetic causes. In hereditary hemochromatosis, there is absorption of a few milligrams of iron each day, in excess of need. As such, clinical manifestations often occur only after the age of 40 years, when body iron stores have reached 15 to 40 g. The symptoms of hemochromatosis are nonspecific and include arthralgia; abdominal pain; fatigue, weakness; impotence; weight loss; amenorrhea and early menopause; abnormal skin pigmentation; damage to the pancreas leading to diabetes; cardiomyopathy; and cirrhosis. The liver is commonly affected with hemochromatosis, and hepatosplenomegaly is commonly seen. There is an abnormal skin pigmentation that is seen with hemochromatosis. In hemochromatosis, the plasma iron will be elevated; total iron binding capacity will be normal or low; and transferrin saturation will be elevated. Hepatocellular carcinoma is the most serious complication, and it is a major cause of death in patients with hemochromatosis. A large percentage of patients with hemochromatosis will develop problems with their pancreas; however, the pancreatic pathology commonly seen with hemochromatosis is insulin dependent diabetes, not pancreatic cancer. a, b, c, e- Bronchogenic carcinoma, pancreatic carcinoma, lymphoma, and leukemia are not known complications of hemochromatosis.

A 43 yo F presents with a 2-year history of worsening dyspnea and a 1-month history of a dry cough. The patient gives no history of fever, chills, chest pain, or wheeze. History is significant for smoking (25 cigarettes/day for more than 22 years). A CXR shows hyperinflated lungs with bullae, tubular heart, flattened diaphragm, and no areas of consolidation. PFTs reveal a decreased FEV1 along with reduction of FEV1/FVC ratio. These findings are characteristic of what condition? a. diffuse alveolar damage b. bronchiectasis c. congestive cardiac failure d. emphysema e. chronic bronchitis

d-Emphysema is a chronic obstructive airway disease characterized by abnormal, permanent enlargement of the airways distal to the terminal bronchioles and destruction of their walls. Cigarette smoking is a major contributor in the development of emphysema. Other causes include air pollution, alpha1-antitrypsin deficiency, and occupational exposure. The classical symptomatology includes prolonged progressive dyspnea with late-onset non-productive cough, occasional mucopurulent relapses, and eventual cachexia and respiratory failure. The patients are usually thin and have a barrel-shaped chest. There is tachypnea with pursed lip breathing and use of accessory muscles; they may also adopt a tripod sitting position. A CXR shows hyperinflated lungs with bullae, tubular heart, and flattened diaphragm. Forced expiratory spirometry quantifies airway obstruction. PFTs reveal a decrease in FEV1 along with reduction of FEV1/FVC ratio. Treatment includes general measures such as cessation of smoking, breathing exercises, symptomatic treatment like antibiotics to control secondary infection, bronchodilators, corticosteroids, and oxygen therapy. Lung volume reduction surgery and lung transplantation may be required depending on the severity. a- Diffuse alveolar damage, or adult respiratory distress syndrome (ARDS), is characterized by fulminant interstitial and alveolar edema that develops after an initial lung injury. It results from increased alveolar capillary permeability that is not cardiogenic in origin. Injury can occur directly or as part of a generalized systemic acute inflammatory process. b- Bronchiectasis is a condition characterized by abnormal permanent distortion of the conducting bronchi or airways, most often secondary to an infectious process. Patient with bronchiectasis often present with chronic cough with mucopurulent sputum production, which lasts months to years. CT, especially HRCT (high-resolution computed tomography), helps in confirming the diagnosis. c- Congestive cardiac failure presents with dyspnea, cough, orthopnea, weakness, abdominal discomfort, and edema in lower portions of the body. The chest X-ray may show cardiac enlargement. There may be pulmonary congestion with audible rales. Echocardiography confirms the diagnosis. e- Chronic bronchitis produces obstructive changes and bronchial inflammation. Chronic bronchitis is characterized by chronic productive cough for at least 3 or more months in each of 2 successive years for which other causes, such as infection with Mycobacterium tuberculosis, carcinoma of the lung, or chronic heart failure, have been excluded. Chest X-ray shows increased bronchovascular markings and cardiomegaly.

A 16 yo M high school football player presents after being tackled and slammed onto his right dominant shoulder forcefully 2 hours ago during a game. He had immediate pain, but was able to continue punting. He has full active and passive range of motion, but some pain (4/10) with abduction. There is no obvious deformity and the skin over the shoulder is intact and not tented. What would you expect to find on a physical exam? a. paresthesia over the elbow b. decreased radial pulse c. crepitus with motion d. tenderness over the greater tuberosity e. tenderness over the AC joint

e- Acromioclavicular (AC) separations typically occur as a result of a direct blow to the very lateral shoulder, usually a fall onto the shoulder. This AC separation is probably a Type I. Type I will have tenderness over the AC joint, no deformity and fair motion with some pain but the patient may still be able to play football with the shoulders padded and limited contact. Pain occurs with abduction mainly. They will typically hold their arm adducted and supported. d- Tenderness over the greater tuberosity would be seen with rotator cuff pathology or proximal humerus fractures. c- Crepitus would be seen with fracture or Type III AC separations. a & b- Nerve and/or vascular injuries are very rare with this injury.

A white female comes to urgent care with a c/o red eyes with a sticky discharge. Her eye feels irritated, the pain is mild. She tells you she shared towels with her boyfriend who had a red eye. She asks you if you think her boyfriend gave her an infection. What is your working diagnosis? a. foreign body b. ectropion c. chalazion d. blepharitis e. conjunctivitis

e- Conjunctivitis can be caused by bacterial, viral, allergic, and irritant etiologies. Clients usually complain of red eyes and a sticky or watery discharge. The type of discharge helps to determine the etiology-watery in viral, and sticky green or yellow, in bacterial conjunctivitis. Irritation is common, but severe pain and photophobia are not. Bacterial or viral conjunctivitis is usually self-limited, but it may be treated with a topical antibiotic, without steroids, such as sulfacetamide (10% 3-4 times/day). Topical aminoglycoside should be reserved for more refractory disease. Allergic conjunctivitis may be effectively treated with a new class of non-steroidal topical anti-inflammatory agents. Irritant conjunctivitis, including dry eyes, may be treated with topical, non-preserved lubricants. a- Foreign Body sensation is most commonly due to corneal or conjunctival foreign bodies. Other causes are disturbances of the corneal epithelium and rubbing of eyelashes against the cornea (trichiasis). b- Ectropion is the outward turning of, usually, the lower lid occurring in older people. Surgery is indicated if ectropion causes excessive tearing, exposure, keratitis, or a cosmetic problem. c- Chalazion is a mildly painful swelling of the eyelid margin, due to granulomatous inflammation, and usually resolves spontaneously with warm soaks and time. Multiple chalazion may be seen in diabetes, alcoholism, and malnourishment. Rarely, curettage may be required. d- Blepharitis may be seborrheic or ulcerative. Seborrheic (non-ulcerative) blepharitis is commonly associated with seborrhea of the face, eyebrows, external ears, and scalp. Inflammation of the eyelid margins occurs, with redness, thickening, and often the formation of scales and crusts, or shallow marginal ulcers. Ulcerative blepharitis is caused by bacterial infection (usually staphylococcal) of the lash follicles and the meibomian glands.

A 41 yo G1P1 presents with a 4-month history of menorrhagia. She is having shortened menstrual cycles that are sometimes only 15 days in length, with menstrual bleeding for 5-6 days. She is using approximately 12-14 pads or tampons per day. She admits to fatigue, headaches, and occasional dizziness, but denies syncope. There is no dysmenorrhea. A thyroid-stimulating test last month was within normal limits. Abdominal and pelvic exams are normal. What statement is true regarding dysfunctional uterine bleeding? a. it is never associated with ovulatory cycles b. the condition is not seen in adolescents c. dilatation & curettage is the only treatment d. it is seen in all age groups equally e. the condition is MC after 40 years of age

e- Dysfunctional uterine bleeding (DUB) is a condition of irregular uterine bleeding in a patient who does not have an anatomic uterine lesion. It is most common above the age of 40 years (50% of the cases), but it is also seen in adolescents (20%), in whom it is associated with anovulatory cycles. Anovulatory cycles are characterized by abnormal levels of estrogen and may be due to estrogen withdrawal or breakthrough. A deteriorating ovarian follicular function is responsible for anovulatory bleeding during the climacteric. Other etiologies, such as polycystic ovarian disease, fibroids, and thyroid disease need to be ruled out before making the diagnosis. An endometrial biopsy or dilation and curettage (if the patient cannot undergo an endometrial biopsy in the office) can be diagnostic, but it is not curative or even therapeutic in a patient with DUB. Medical therapy, including estrogens, progestational agents, progesterone-impregnated intrauterine devices, and combination oral contraceptives are used to treat the condition.

A 14 yo M develops bloody diarrhea with abdominal cramping. Double contrast barium enema shows fine serrations and narrowing of the rectum and sigmoid. Stool contains mucus, blood, and white blood cells, but no parasites or bacterial pathogens. Endoscopy shows inflamed mucosa and pseudopolyps. Biopsy finds extensive inflammatory process in the mucosa and submucosa. Glands are filled with eosinophilic secretions. There is also mild involvement of the terminal ileum. Sulfasalazine treatment was attempted but failed to bring improvement. What is the most appropriate next step in the management? a. diphenoxylate b. loperamide c. 6-mercaptopurine d. metronidazole e. corticosteroids

e- Fine serrations (ulcers) and pseudopolyps suggest ulcerative colitis. It is usually limited to the colon but occasionally, as in this case, the terminal ileum is also affected ("backwash" ileitis). In addition to sulfasalazine or 5-amino-salicylacid, more severe ulcerative colitis cases require corticosteroids. Corticosteroids are also helpful in Crohn's disease involving the small bowel. While surgical removal of the colon can give relief in ulcerative colitis, Crohn's disease tends to recur in about half of the cases involving removal of the diseased bowel. a & b- Diphenoxylate and loperamide provide only symptomatic relief. c- Immunosuppressive agents, like with 6-mercaptopurine, are used in severe Crohn's disease cases. d- Metronidazole is helpful in treating the characteristic fistulas of Crohn's disease.

A 48 yo F presents with a 2-month history of heartburn, epigastric discomfort, nausea, and occasional vomiting. She has a history of hyperlipidemia, controlled with diet and exercise, as well as asthma, for which she takes inhalers as needed. She takes no other medications, including OTC analgesics. Family history is noncontributory. On exam she is afebrile, BP120/70 mm Hg, pulse 74/min, and SPO2 92%. Lungs are clear, and she has minimal epigastric tenderness. Otherwise, physical examination is unremarkable. She is advised by her physician to take lansoprazole once daily, which provides only partial relief. Endoscopy is then recommended, which shows a duodenal ulcer. Biopsy reveals infection with Helicobacter pylori. What would be the recommended regimen at this time? a. bismuth, metronidazole, & tetracycline 4 times daily for 2 weeks b. bismuth, metronidazole, & lansoprazole twice daily for 2 weeks c. omeprazole, tetracycline, & clarithromycin twice daily for 2 weeks d. lansoprazole, amoxicillin, & metronidazole twice daily for 2 weeks e. pantoprazole, amoxicillin, & clarithromycin twice daily for 2 weeks

e- The choice of the regimen depends on considerations such as cost, side effects, and ease of administration. Allergy to one of the medications, as well as intolerance, should also be taken into account. Any PPI with amoxicillin 1000 mg twice daily and clarithromycin 500 mg twice daily for 2 weeks or PPI with metronidazole 500mg twice daily and clarithromycin 500mg twice daily for 2 weeks are recommended. These are the triple drug therapies available. The other regimens suggested are bismuth, metronidazole, and tetracycline 4 times daily for 2 weeks along with PPI twice daily for 2 weeks or H2 receptor antagonist twice daily for 4 weeks (quadruple drug therapy). Dual therapy with a proton pump inhibitor and an antibiotic (amoxicillin or clarithromycin) is not recommended as primary therapy, since eradication rates are much lower than the above regimens. The most common side effect is a metallic taste in the mouth due to clarithromycin or metronidazole. Amoxicillin can cause diarrhea or a rash. Clarithromycin can also cause nausea, vomiting, abdominal pain, and (rarely) QT prolongation. Metronidazole can cause peripheral neuropathy, seizures, and a disulfiram-like reaction when taken with alcohol. Tetracycline is teratogenic and causes photosensitivity.

A 73 yo M presents with a 4-month history of DOE and a productive cough. He also unintentionally lost 10 pounds in 2 months. His PMH is significant for coronary artery disease and a myocardial infarction. He has smoked an occasional cigar the last few years. He has been retired for 12 years, but for 30 years he worked odd jobs in the construction industry. He also helped his father in the family's car garage shop. Vital signs are normal. His physical exam is remarkable for decreased breath sounds in left lower lung fields and dullness to percussion. A chest radiograph is ordered, and it shows a left sided pleural effusion. What condition does this patient have? a. recurrent postmyocardial infarction pericarditis b. pneumonia c. CHF d. lung cancer e. malignant mesothelioma

e- His work history predisposed him to asbestos. Mesothelioma can take more than 40 years to surface, and the most common industries that expose people to asbestos are construction, mining, brake linings, roofing, and working in shipyards. This patient worked in both construction and a car garage shop, which would have likely exposed him routinely to asbestos a- this is incorrect because there was no mention of pleuritic or inspiratory chest pain b- Even though he could have this as an underlying disease, the pleural effusion seen on the chest radiograph is indicative of malignant mesothelioma c- Congestive heart failure is an incorrect choice; his work history is the most important thing to focus on in this case. There is no mention of any significant cardiac findings on exam. Also, there is a history of loss of weight, which favors mesothelioma d- Lung cancer is not likely as he does not have the classic risk factors for developing it. His cigar smoking history is not significant

A 77 yo M presents to your clinic c/o an acute onset of severe periumbilical pain with nausea and vomiting. PMH is remarkable for CAD with atrial fibrillation. Physical examination is remarkable for minimal abdominal distension and guaiac positive stool. The suspected diagnosis is acute mesenteric ischemia. Which is the gold standard diagnostic imaging study to confirm the suspected diagnosis? a. Doppler US b. colonoscopy c. CT d. plain radiography e. angiography

e- In patients with acute or chronic intestinal ischemia, a CTA or MRA can demonstrate narrowing of the proximal visceral vessels. In acute intestinal ischemia from a nonocclusive low flow state, angiography is needed to display the typical "pruned tree" appearance of the distal visceral vascular bed.

A 66 yo F with a long-standing history of HTN presents with a severe headache; it started this morning and is rapidly worsening. During the interview, she suddenly collapses. Your brief examination shows that she responds with extensor posturing on external stimuli. Her deep tendon reflexes are 3+, and you elicit Babinski bilaterally. You also notice that her breathing has a peculiar pattern: deep inspiration with a pause at full inspiration, followed by a brief insufficient release and the end-inspiration pause. How do you best describe her respiratory pattern? a. cluster b. ataxic c. Kussmaul d. Cheyne-Stokes e. apneusis

e- In the hypertensive patient with sudden loss of consciousness and decerebrate response (extensor posturing), you should consider brain stem hemorrhage. Abnormal breathing patterns can be observed in both pontine and medullary lesions; they sometimes can be prognostic. Her breathing pattern is apneustic. Apneustic breathing pattern characterizes deep, gasping inspiration with a pause at full inspiration followed by a brief, insufficient release and the end-inspiration pause before expiration. Lesion in the pons or upper medulla causes the removal of input from the vagus nerve and the pneumotaxic center. Normally, the apneustic center of the lower pons promotes inspiration by stimulation of the dorsal respiratory center in the medulla to delay the 'switch off' signal of the inspiratory ramp provided by the pneumotaxic center of pons. Therefore, the apneustic center controls the intensity of breathing. Apneusis is an ominous sign, with a generally poor prognosis. a- Cluster breathing is irregular breathing with periods of apnea that occurs at irregular intervals. It is generally seen in lesions in the low pons or upper medulla. It differs from Cheyne-Stokes pattern because there is no increasing and decreasing depth of respirations. b- Ataxic breathing (Biot's respiration) is an abnormal pattern of breathing characterized by groups of quick, shallow inspirations followed by regular or irregular periods of apnea. It is generally a poor prognostic sign. Biot's respiration is caused by the medullary lesion due to strokes or trauma or by pressure on the medulla due to uncal or tentorial herniation. c- Kussmaul breathing is a deep, labored, and gasping breathing pattern seen often in severe metabolic acidosis (diabetic ketoacidosis, renal failure). In metabolic acidosis, breathing is first rapid and shallow, but later on, as acidosis worsens, breathing gradually becomes Kussmaul breathing. d- Cheyne-Stokes respiration is an abnormal pattern of breathing characterized by repetitive progressively deeper and sometimes faster breathing, followed by a gradual decrease in breathing that results in temporary apnea. Cycles usually take 30 seconds to 2 minutes. Increased CO2 during the period of apnea causes compensatory hyperventilation. Hyperventilation in turn causes the decrease in CO2, which causes apnea and the cycle to restart. Causes include CNS dysfunction, cardiac failure with low cardiac output, sleep, hypoxia, or profound hypocapnia.

A recognized complication of sleep apnea is ____________. a. migraines b. diabetes mellitus c. RLS d. hyperlipidemia e. CHF

e- Obstructive sleep apnea occurs most often in moderately or severely obese persons. Men are affected more often than women (4% of men and 2% of women in middle age). Upper airway narrowing leads to obstruction during sleep. In severely obese persons, a combination of hypoxemia and hypercapnia may induce central apnea as well. By definition, apneic periods last at least 10 seconds (some for 2 minutes). Repeated nocturnal obstruction may cause recurring cycles of sleep, obstructive choking, and arousal with gasping for air. Daytime drowsiness usually results from the repeated cycles. Similar but less-pronounced cycles occur in nonobese persons, possibly secondary to developmental or congenital abnormalities of the upper airway. Complications of sleep apnea include cardiac abnormalities (e.g., sinus arrhythmias, extreme bradycardia, atrial flutter, ventricular tachycardia, heart failure), hypertension, excessive daytime sleepiness, morning headache, and slowed mentation. The mortality rate from stroke and MIs is significantly higher in persons with obstructive sleep apnea than in the general population.

A 69 yo F presents saying her husband says she doesn't listen to anything he says. The patient states that occasionally she has to ask people to repeat themselves when sitting to her right. She denies any dizziness, headaches, or visual disturbances. Her current medication is furosemide. On physical examination, the Weber test reveals lateralization to the left ear. Air conduction lasted for 15 seconds and bone conduction lasted 10 seconds. What do you suspect as the cause of this hearing loss? a. middle ear effusion b. cerumen impaction c. Meniere's disease d. otosclerosis e. ototoxicity

e- Ototoxicity secondary to furosemide is the correct answer. Loop diuretics can cause sensory hearing loss, as evidenced by this patient's physical exam finding of lateralization to the good ear and air conduction slightly longer than bone conduction. The Rinne test should reveal an air:bone conduction ratio of 2:1. a- Middle ear effusion is incorrect because it would cause a conductive hearing loss. b- Cerumen impaction will cause a conductive hearing loss with the lateralization to the affected ear and a negative Rinne test. c- Meniere's Disease is incorrect because the patient does not exhibit any vertigo or tinnitus. d- Otosclerosis typically will result in conductive hearing loss.

A 5 yo girl has paroxysms of cough that increase in severity and duration. Some coughing episodes are followed by a whooplike, high-pitched inspiratory noise, and vomiting has also occurred after paroxysms. What laboratory test could lead to the earliest confirmation of the likely diagnosis? a. immunofluorescent antibody staining b. CXR c. culture d. blood cell analysis e. PCR assay & antigen detection

e- The characteristics presented by this child suggest B. pertussis or parapertussis infection. The diagnosis of pertussis is still primarily clinical, and laboratory results only play a small role in the diagnostics. b- Chest X-ray cannot confirm the suspected diagnosis. d- WBC count is likely to be elevated, with the lymphocytes composing 70-80% of the total count. c- The infection can be directly confirmed by culture for Bordetella pertussis, but it takes several days, and there is a high rate of false negative results. a- Immunofluorescent staining of nasopharyngeal washing can be done, and it is significantly more sensitive than culture. Although culture used to be the gold standard, PCR assay and antigen detection are now considered more reliable. PCR assay and antigen detection are increasingly used to assist in diagnosing pertussis. Advantages include greater sensitivity, more rapidly available results, and use later in the disease course or after antimicrobial therapy because the tests do not rely on the isolation of viable organisms. Their use is limited by lack of standardization and incomplete understanding of the correlation between test results and the course of the illness.

A 64 yo M with a PMH significant for a 15-year history of HTN presents with severe tearing chest pain radiating through to the back. His blood pressure is 180/110 mm Hg, heart rate is 120 BPM, and respiratory rate is 34/min. Physical exam findings include neck negative for bruits/jugular venous distension (JVD), lungs clear to auscultation, heart regular rhythm despite tachycardia, normal S1/S2 with an S4 present, and a grade III/IV diastolic rumbling murmur noted with patient leaning forward. Radial pulses are 1+ on right and 3+ on left. EKG reveals a sinus tachycardia and evidence of left ventricular hypertrophy. What is the most likely etiology of this patient's symptoms? a. coarctation of the aorta b. spontaneous pneumothorax c. acute viral pericarditis d. pulmonary saddle embolus e. thoracic aortic dissection

e- The clinical picture is suggestive of a thoracic aortic dissection. This condition is most common in men between 60 and 70 years of age. On physical examination, peripheral pulses and blood pressures may be increased, diminished, or unequal. A diastolic murmur of aortic insufficiency may be heard. a- With coarctation of the aorta, there can be variations in blood pressures based on where the defect has occurred along the aorta. The most common is an elevation of blood pressure in the upper extremities compared to the lower. Rib notching on chest X-ray is a general finding with this condition. b- Spontaneous pneumothorax can be classified as primary (absence of an underlying lung disease) or secondary (complication of a preexisting pulmonary disease). Primary mainly affects tall, thin boys and males between the ages of 10 and 30 years. Family history and cigarette smoking may be important factors. c- With pericarditis, the patient does present with chest pain, but it is usually described as sharp, retrosternal pain with radiation to the back that worsens with deep breathing or coughing. The pain is position-dependent, worse when lying flat, and improved while sitting up and leaning forward. On physical examination, the pericardial rub is pathognomonic of pericarditis. d- A saddle emboli occurs at the level of the bifurcation of the pulmonary trunk, extending into the main right and the main left pulmonary arteries. No single symptom or sign or combination of clinical findings is specific to pulmonary emboli. Some findings may include dyspnea and pain on inspiration, as well as tachypnea.

A 31 yo M presents with SOB, wheezing, mild fever, and fatigue. He has had several similar episodes in the past, and each previous episode began after a cold that moved into his chest. Over the past several weeks, he has had a productive cough most mornings. He smokes on a social basis. What is the most likely diagnosis? a. bronchiectasis b. chronic bronchitis c. chronic emphysema d. cor pulmonale e. acute asthmatic bronchitis

e- The clinical picture is suggestive of acute asthmatic bronchitis. Typical symptoms include persistent cough with mucus that can become thicker and more profuse, dyspnea, mild fever, and chest pains. It is commonly caused by a virus infecting the lining of the bronchial tree. Wheezing can occur for several weeks. In this particular patient, the cold (viral infection) moved to his lungs and induced acute asthmatic bronchitis. a- Bronchiectasis presents with dyspnea, wheezing, chronic cough with copious mucous production, hemoptysis, and pleuritic chest pain. Hemoptysis and pleuritic chest pain are not present in this patient. b- Chronic bronchitis is defined as a clinical history of productive cough for 3 months of the year for 2 consecutive years, which is not described in this patient. c- Productive cough is not seen in chronic emphysema patients. d- Cor pulmonale has similar symptoms to bronchitis and emphysema, but it also has elevated jugular venous pressures, parasternal lift, edema, hepatomegaly, and ascites, which are not seen in this patient.

A 49 yo M presents to your office for a follow-up appointment of his hypertension. He has complaints of some non-specific chest discomfort, so you decide to perform an ECG. The ECG demonstrates peaked T waves in several leads without any other abnormality. Which of the following medications is most likely to cause this ECG finding? a. prazosin b. hydrochlorothiazide c. atenolol d. furosemide e. lisinopril

e- The correct answer is lisinopril, as it is an angiotensin converting enzyme inhibitor that blocks the production of aldosterone, leading to possible hyperkalemia. Peaked T waves on ECG are indicative of hyperkalemia. d- Furosemide is a loop diuretic and is associated with hypokalemia. c- Atenolol is a beta-blocker and is not associated with hyperkalemia. b- Hydrochlorothiazide is a thiazide diuretic and may cause potassium loss. a- Prazosin is an alpha-blocker antihypertensive and is not associated with hyperkalemia.

A 53 yo F living a non-sedentary lifestyle presents with a 5-day history of low-grade fever, flu-like syndrome, sore throat, and malaise. She is mostly bothered by the fact that she has to "catch" her breath because of pain on inspiration and when coughing. She has no known past medical or surgical history; she is not on any medication, and she has no pertinent family history. She denies any medication use, including OTC medicines. On physical examination, her vitals are: Temperature 100.6 F, pulse 86/min, BP133/75 mm Hg, and RR 20 breaths/min. She has shallow breathing, resonant percussion notes, fair air entry with vesicular breath sounds, and friction rub. Her blood gas on room air is: pH 7.36, PCO2 44 mmHg, PO2 100 mmHg, HCO3 26 mEq/L, O2 saturation 99%. Her CXR is normal and the D-dimer assay is also normal. What is the most appropriate management modality for this patient? a. IV heparin b. reassurance c. V/Q scan d. CT scan e. NSAIDs

e- The patient has pleuritis without effusion. Pleuritis is inflammation of the pleura. It may occur with or without effusion and is characterized by a sharp pain worsened by cough and inspiration. The pain may radiate to the shoulder if the diaphragmatic pleura is affected and pressure around the area of inflammation may produce pain. Friction rub is heard on auscultation. The pleuritic chest pain causes shallow rapid breaths. The presentation with flu-like illness, malaise, absence of history of collagen vascular disease, and CXR findings makes the most likely etiology viral in this case. Pleuritis is treated with NSAIDs. Ibuprofen can be given in the dose of 400 - 800 mg orally 3 times daily for pain management in case of pleurisy due to viral etiology. Other examples of etiologic agents are rheumatoid arthritis, systemic lupus erythematosus, radiation, pneumonia, acute pulmonary embolism, and pneumothorax. c & d- Although pulmonary embolism can cause pleuritis, the D-dimer assay is normal in this patient; no further evaluation by imaging modalities, such as V/Q scanning or CT scanning, is warranted in a person at low risk of a pulmonary embolism. Common risk factors are prolonged immobilization, congestive heart failure, stroke, pelvic fracture, obesity, and malignancy. Patients present with sudden shortness of breath, tachycardia, tachypnea, hypotension (if massive), and low-grade fever. There is hypoxemia on blood gas, and CXR may be normal or show a region of oligemia and wedge-shaped density. Treatment is by anticoagulation.

A 27 yo F presents with fatigue. She has been diabetic for 10 years, and she takes multiple injections of insulin a day. She has no other medical problems. She takes no other medications, and she is not sexually active. Her skin shows diffuse scratch marks. Her laboratory tests show: Hemoglobin 14g/dl Total white count 5.0 X 10^3 Spot urine albumin/creatinine ratio 50 mg/g Urinalysis ++ protein, otherwise normal Glomerular filtration rate 60 ml/min/1.73 m2 Calcium 7.0 mg/dl Albumin 3.9 g/l Phosphorus 6.0 mg/dl 25(OH)D <30 ng/ml PTH 150 pg/ml Bicarbonate 23meq/l Potassium 4.5meq/l Sodium 139 meq/l What is the most likely diagnosis? a. hypercalcemia b. hypervitaminosis D c. hypervitaminosis A d. hypophosphatemia e. secondary hyperparathyroidism

e- This patient has stage 3 kidney disease, based on her GFR of 60 ml/min/1.73 m2 and her proteinuria. It is also likely that she has hyperparathyroidism. Disorders of calcium and phosphorus balance are common in kidney disease and should be evaluated starting early in the course of disease to prevent complications. Vitamin D levels, calcium and phosphorus levels, and parathyroid hormone levels (PTH) should be checked in all patients with stages 3 - 5 chronic kidney disease. Decreasing GFR leads to phosphorus retention. Decreased renal synthesis of hydroxylated vitamin D leads to decreased intestinal calcium reabsorption and hypocalcemia. Together, hyperphosphatemia, hypocalcemia, and hypovitaminosis D lead to hyperparathyroidism. Vitamin D usually suppresses PTH synthesis and secretion. Vitamin D deficiency removes this negative feedback, leading to hyperparathyroidism. Correction of calcium-phosphate balance is important; when it is abnormal, it can lead to vascular and valvular calcification and increased risk of cardiac death. Her phosphorus level is elevated, not low. Goal levels for patients with stages 3 and 4 kidney disease are 2.7 - 4.6 mg/dl. Her calcium level is low, not elevated. Her corrected level should be >8 mg/dl. Her PTH level is 150. Patients with stage 3 should have levels in the 35-70 pg/ml range (Brenner, ch. 52), according to National Kidney Foundation guidelines. No mention is made of Vitamin A levels in this vignette. Her vitamin D levels are low, which is consistent with impaired hydroxylation of the vitamin D compound by her kidneys. Patients with levels <30 ng/dl should be supplemented with ergocalciferol. Hypervitaminosis D (Vitamin D intoxication) occurs in the accidental or intentional intake of vitamin D or vitamin D compounds including vitamin D, vitamin D fortified milk, etc. High levels of vitamin D would be noted in this case. Additionally, since Vitamin D suppresses PTH, low levels of PTH would be noted. Vitamin D causes increased gastrointestinal calcium absorption and increased serum calcium levels. Symptoms are those of hypercalcemia (bone and abdominal pain, weakness, confusion, and possibly hematuria/flank pain/poor urine flow if renal stones develop). Treatment involves promoting kaliuresis with volume expansion and possibly loop diuretics and steroids. a- Correction of calcium-phosphate balance is important; when it is abnormal, it can lead to vascular and valvular calcification and increased risk of cardiac death b- Her vitamin D levels are low, which is consistent with impaired hydroxylation of the vitamin D compound by her kidneys. Patients with levels <30 ng/dl should be supplemented with ergocalciferol. Hypervitaminosis D (Vitamin D intoxication) occurs in the accidental or intentional intake of vitamin D or vitamin D compounds including vitamin D, vitamin D fortified milk, etc. High levels of vitamin D would be noted in this case. Additionally, since Vitamin D suppresses PTH, low levels of PTH would be noted. Vitamin D causes increased gastrointestinal calcium absorption and increased serum calcium levels. Symptoms are those of hypercalcemia (bone and abdominal pain, weakness, confusion, and possibly hematuria/flank pain/poor urine flow if renal stones develop). Treatment involves promoting kaliuresis with volume expansion and possibly loop diuretics and steroids c- No mention is made of Vitamin A levels in this vignette. d- Her phosphorus level is elevated, not low. Goal levels for patients with stages 3 and 4 kidney disease are 2.7-4.6 mg/dl.

A 38 yo M presents with nasal stuffiness, headache, fatigue, facial pain, and chronic post-nasal drip. He has had similar episodes in the past; on average, they have occured 2-3 times a year for the last several years. He has been diagnosed with acute sinusitis, and antibiotics have been prescribed; they have provided him with relief for a brief period. This time, however, his symptoms have bothered him on and off for the last 3 months; he was given a 14-day course of antibiotics, but he experienced only partial relief. He is tired of the recurrent episodes, and he wants a cure. On exam, he is afebrile; nasal mucosa is inflamed, and there is mucopurulent secretion in the nasal cavity. The right maxillary sinus is tender on palpation. Lungs are clear. What is the best next step in the management of this patient? a. MRI of the sinuses b. nasal & sinus cultures c. plain X-ray views of the sinuses d. a 3-week course of antibiotics e. CT scan of the sinuses

e- This patient is suffering from chronic sinusitis. Repeated regimens of different antibiotics have not provided him relief, and now he fits the criteria for chronic sinusitis, including 12 weeks of symptoms. Chronic sinusitis is most commonly caused by streptococcus pneumoniae, Haemophilus influenzae, and moraxella catarrhalis. Together, these 3 bacteria account for 70% cases. A limited CT scan of the sinuses defines the location and extent of disease and helps in deciding further management. It is quick, low cost, and sensitive. CT scanning also helps in delineating anastomotic blockage of the osteomeatal complex; therefore, there is a role for it in cases of endoscopic surgery. a- MRI of the sinuses tells us more about the soft tissue pathology, but bony structures cannot be studied in detail. MRI is done if malignancy is suspected or there are signs and symptoms of possible intracranial extension. b- Nasal cultures can be contaminated with colonized organisms in the nose, such as staphylococcus aureus, and do not correlate well with culture obtained from the sinuses. Endoscopically-guided cultures of secretions in the middle meatus or within a sinus are usually not done in clinical practice, even though occasionally it may provide the exact causative pathogen. c- Plain X-rays are no longer recommended; they are not sensitive enough in the visualization of the sinuses, and they often miss findings. d- A 3-week course of antibiotics may relieve symptoms briefly, but it is unlikely to cure him, especially since he already has had several courses. Amoxicillin-clavulanate or cefuroxime are traditionally used for 3 weeks. In intractable cases, a 6-week course may also be given. Clarithromycin and clindamycin are used for patients who are allergic to penicillin. Quinolones are only used if cultures show Gram-negative bacteria.


Ensembles d'études connexes

//AP WORLD// Unit 4 practice questions + explanations

View Set

Everything, Everything(Complete)

View Set

Cognitive Psychology Final Exam - Dr. Lisa Topp-Manriquez (Unit 3 Review)

View Set

MGMT 3000 Simoon Cannon Exam 1, Managment test 1, intro to organizational management Ch. 4-7

View Set

Chapter 5: Conquest and Exile: A Remnant Returns

View Set

OCCUPATIONAL LUNG DISEASES - PNEUMOCONIOSES

View Set